Download as pdf
Download as pdf
You are on page 1of 103
Transmission Lines and Matching Networks 8.1, INTRODUCTION &L IS USUALLY necessary in various communication systems to interconnect points which are some distance apart from each other. For example, the connection between a transmitter and its antenna or antenna and receiver. The intermediate distance between two points may become an appreciable fraction of wavelength being propagated, at high frequency. It, thus, becomes essential to consider the properties of the inter-connecting wires, as they do not behave as short circuits. Evidently the separation between wires, their sizes, lengths and general layout must be studied under these conditions. In the applications of antennas, it is necessary to employ transmission lines either in some form or the other form to connect a transmitter or a receiver. Therefore, it is also necessary to know the propagation characteristics of the commonly used transmission lines and the devices constructed from the transmission lines, The knowledge of transmission lines (or feeders) in the study of antennas are essential for the following three reasons : (i Transmission lines (or feeders) are almost always used for connecting a transmitter and its antenna (or antenna and receiver) and it is usually regarded as a part of the antenna system, Gi) Transmission lines elements are integral parts of the antenna, in some antenna system, and (iii) Transmission lines’ basic principles are applicable in understanding some aspects of antenna theory e.g. definition, current and voltage distribution etc. Besides, a feeder (as used in antenna applications) is a particular case of a transmission line. However, in case of feeders, it is customary to use certain approximate formulae which are entirely satisfactory for the problems presented by feeders but are not always applicable to all the transmission line problems. TRANSMISSION LINES AND MATCHING NETWORKS 609 Inthis chapter, . se empnisia oA ee en the basic aspects of transmission line theory will be discussed with t ipplications. Transmission lines deserve its own right to be dealt with in details ee Nees accion ere may be referred to. Since a transmission line is used to transmit energy pone place to another, butb ides this, itcan also be used as circuit elements like inductors, capacitors, reso » filters, transformers and even insulators at very high frequencies. Accordingly, some of its uses in matching e.g. stub matching, quarter wi ens +, ave transformers, balance to unbalance (bali) transformer and directional couplers will also be studied in this chapter: os 82. BASIC PRINCIPLES OF TRANSMISSION LINES Transmission lines are a means to convey electrical signals or power between two points separated appreciably in distance. It is a specific example of linear passive four terminals network or a linear passive two port network of circuit theory. The simplest type of a transmission line, is a pair of parallel wires insulated from each other as shown in Fig. 8.1. It is also known as two-wire balanced transmission line, The basic principles are studied in terms of this type of transmission line. However, they are also applicable to other types of transmission lines as such or with modifications. + ‘s $F HUTPUT OR ze Jreceyync| jw a TERMINALS. Seats Fig. 8.1. Two wire (balanced) transmission lines and its dimensional elements. Vs = Sinusoidal input voltage. Zs = Input impedance. d = 2r = diameter of each conductor. Ze = Load impedance. 5 = Spacing between them. Subscript S = for sending or input end. 1 = length of the line. Subscript R = Receiving end or load end ‘The diameter d, the spacing S, the length /, are dimensional properties of the line and are important electrically. In addition, electrical properties e.g. conductivity of wires, dielectric constant, permeability, loss characteristics of any material within the field of the line are also important. For the operation, a load impedance Ze,-is connected at the load terminals and a sinusoidal voltage Vs is applied to the input terminals of the line: Now the problem inthe transmission lin is to find out the current and voltage along the line (at any point) and atthe load, if Vs and Ze are given. Further the input impedance Zs is also to be known, The input impedance Zs, by definition is the ratio of input voltage Vs tothe input current Is and is easily calculated if Vs and /s is determined. Input impedance is the impedance that transmission line presents at its input terminals to the source of input voltage. ‘A transmission line electrical problem differs from the simple four terminal network of circuit theory in one aspect i.e. as the input voltage Vs is applied at the input terminals, it takes a finite time in reaching to load end terminals. Therefore; firte time is needed forthe voltage and current to travel the Tength of the transmission line just as the electromagnetic wave has a finite velocity in space. Truely speaking voltage and current travelling along are accompanied by an electromagnetic wave in space between conductors and transmission line problem is analysed in terms of this field rather in terms of voltage and current. The transmission line can, then, be regarded as a means of guiding the field so that it is confined near the line rather than spreading in space, spherically. As the wave is confined, it does not suffer inverse-square law decrease of power-density with distance, unlike waves in free space. The elecirical elements of the transmission line problem is shown in Fig, 8.2. $0 ELECTROMAGNETIC FIELDS AND WAVES safpiwe RECEIVING END END Zz Fig. 8.2, Electrical elements of the line. 8.3. EQUIVALENT CIRCUIT REPRESENTATION OF TRANSMISSION LINE. Just as an ordinary four terminal or two port network, a transmission line has inductance (L), capacitance (C) and resistance (R). The essential difference between them is that these electrical Parameters are uniformly and continuously distributed along the line while in ordinary circuits they are lumped, ice each conductor of the line has a certain lengtii and diameter, it must have resistance (R) and inductance (L). Further, since the two wires or conductors are close to each other, therefore, there must be capacitance (C) between them. Lastly, since the wires are separated by a medium popularly known as dielectric that can not be perfect in its insulation and hence current leaks through it. This leakage of current through the dielectric is represented by a shunt conductance (G). The resulting equivalent circuit of a —+| O80 WH Bay. ae Be Be lew Loe Fig. 8.3. General equivalent circuit representation of a transmission line. transmission line is shown in Fig. 8.3 and each identical section in this representation corresponds to a unit length of the transmission line. Here it is assumed that the unit of length is very small fraction of the total line length so that there will be a number of identical lumped unit length sections. One unit section of line is represented upto x = 1 per unit length may be per meter or per kilometre, When R, L, G, Care uniformly distributed along the entire length of a transmission line, itis said to be a uniform transmission line. R, L, C, G should not be assumed to be lumped at any one point as they are ‘uniformly and continuously distributed along entire length of the line. L = Inductance per unit length of the conductors, (i.e. Henrys per metre). C = Capacitance between the conductors per unit length, (i.e. Farad per metre). R = Resistance (series) of conductors per unit length, (ie. ohm per metre) G = Shunt or leakage conductance per unit length, (i.e. Mhos per metre). At radio frequencies (RF), the inductive reactance is much larger than the resistance and so also the capacitive susceptance in comparison to the shunt conductan: R and G both are neglected. This results in a line which is considered at RF are simplified. The simplified equivalent circuit representation i ice. Hence at radio frequencies to be lossless and thus calculation is also shown in Fig. 8.4. b To TY Fig. 8.4. RF equivalent circuit representation ofa transmission line TRANSMISSION LINES AND MATCHING NETWORKS a In order to d hich are not f tly lescribe the behaviour of transmission line, it is helpful to consider two ideal cases wm ully attainable in practice but may be closely approximated. ® Infinite length (1 — © ) of transmission line, and (ii) Lossless transmission line (in which R = 0, G = 0). 84, PRIMARY CONSTANTS OF A TRANSMISSION LINE line and are defined Gees ametes €8. R, La G, C are refered to as primary constants ofthe transmission (® Resistange (R) is defined as loop resistance per unit length of line, It is sum of resistance of both the wires for unit line length and its unit is ohm/km. . (ii) Inductance (L) is defined as loop inductance per unit length of the line. It is sum of inductance of both wires for unit line length and its unit is henries/km. Gi) Conductance (G) is defined as shunt conductance between the two conductors per unit length of line and its unit is Mhos/km. (iv) Capacitance (C) is defined as shunt capacitance between two conductors per unit length of line and its unit is farad/km. Since G and C are present betweer two wires so the loop notation is not used. Although R, L, G, Care referred to as primary constants but in general they vary with frequency. However for y gs y vary y transmission line theory they are assumed constant. ‘The series “mnpedance (Z) and shunt admittance ¥ of the transmission line per unit length are given as (Fig. 8.3) \ Z=R+jol (8.1) Y¥=G+joc (8.2) ‘As will be seen the square root of ratio of Z to ¥ and square root of product of Z and Y will, respectively, give, characteristic impedance and propagation constant 85. BASIC TRANSMISSION LINE EQUATIONS ; ‘The transmission line, here shall be analysed in terms of voltage and current and the relation between voltage ard current ofa line ean be studied by considering te simplest type of transmission line eg a pair of parallel wires, of uniform size, spaced a small distance apart in air, Let us consider short swetion of transmission line PQ of length dx, at a distance «from the sending end A. Tf the length (. impedance Zp is given by Vets a a 1 V= Ik Ze (8.21) Q t and the general solutions of the line terminated with any a impedance Zp at x = ! is given by eqn. 8.20. (Putting ne ast Fig. 8.7. Transmission line terminated = = I)as V = Ve and | = Jpand x =!) with any load impedance Zp TRANSMISSION LINES AND MATCHING NETWORKS 615 Ve = Vs cosh Pl — Is Z sinh Pl (8.22 (@)] te = Is cosh Pt ¥ sinh Pt 8.22 (0)] Putting these eqns. in eqn. 8.21 we have (Vs cosh PI -ZgJs sinh Pl] = Zp [+ cosh Pl — z sinh | or Multiplying both sides by Zo, Z [Vs cosh Pl — Is Zo sinh Pl} = Zp [Is Z cosh Pl — Vs sinh Pl] or Vs [Zo cosh Pl + Zp sinh Pl] = I, Z [Zp cosh Pl + Zp sinh Pl] ee Vs _ 5 [Ze cosh Pl + Z sinh Pl} Is ~ ™ [Zpcosh Pl + Ze sinh PI] _____ Hence the sending end impedance or input impedance of the line as viewed by the source is given by 25 = VE = 7, | Zacosh PL+ Z sinh PI Is Zo cosh Pl + Zp sinh Pl ‘Thus the sending end impedance Zs (or input impedance of the line ) Zg cosh PI + Zo sinh Pl Zo cosh Pl + Zp sinh PL ‘This is the input impedance of a transmission line and is completely general. This eqn. is important and is applicable to all uniform transmission lines and terminations. ‘As a special case, it is seen that if the line is terminated by its characteristic impedance i.e. Zp = Zo, then eqn. 8.23 reduce to Zs=Z {8.23 (a)] _ 7, | 2 cosh Pl + Zo sinh Pl Bat [ ‘Z cosh Pl + Zo sinh Pl | Zs = ZH (8.24) Thus, the input impedance of a finite transmission line terminated in its characteristic impedance Z, is the characteri: impedance of the line. Eqn. 8.23, if divided by cosh PI num. and deno., will give, the eqn. of input impedance in an alternative form ¢-8, or Za + Z tanh Pl ts = Bo s + Zp tanh Pl -+{8.23 (0)] . VOLTAGE AND CURRENT AT ANY POINT ON A TRANSMISSION LINE oe Vth general equations are V = Vs cosh Px ~ Is Zo sinh Px 18.20()] and 1 = Is cosh Px - om sinh Px [8200] “These expressions involve /s which can be determined only if Zsis known. This disadvantage can be removed if Is is replaced by Vs/Zs and using the expression for Zs . Since Po Vs “Zs If the line is of length, terminated by load impedance Zp Is 616 ELECTROMAGNETIC FIELDS AND WAVES Vs Ys Zs Ig = Ys { Zo cosh Pl + Ze sinh PL ** Ze | Ze cosh Pl + Z sinh Pl Putting this eqn. in eqn. 8.20 (a), we get 22 cosh Pl + Zp antl} (8.25) Z| Zr cosh Pl + Z sinh Pl Zo sinh Px x Vs [ Zocosh Pl + Zp sinh Pl ] DB Ze cosh Pl + Z sinh Pl = Vs cosh Px x [Ze cosh Pl + Zp sinh Pl] Vs sinh Ps x [Z cosh Pl.+ Ze sinh Pt] [Ze cosh Pl + Zp sinh P} y= Vs lcoshPx x Ze coshPl + cosh Px x Zp sinhPl] - Vs [sinhPx x Zo coshPl + sinh P; x Zr sinhPx] [Ze cosh Pl + Zp sinh P} Vs [Zp cosh Px cosh Pl —- Zp sinh Px sinh Pl] + Vs [ Zo cosh Px sinh Pl — Zo sinh Px . cosh Pl} [Zo cosh Pl+ Zp sinh PL] _ Vs (Zecosh P(I — x) + Z sinh P(1 — x or |v = 818 [Ze cosh Pl . Prats 4 {8.26 (a)] Similarly, if we substitute the value Is into eqn. 8.20 (b), we get the value of current as. 1-#| 2 Ses | 18.260) h Zz cosh Pl + Zo sinh Pl V = Vs cosh Px ~ Zp sinh Px { V = Vs cosh Px ~ or or V= These eqn. 8.26 (a, b) give the value of voltage and current at any distance x measured from the sending end in a line of length J. 8.9. VOLTAGE AND CURRENT WHEN LINE IS OPEN CIRCUITED AT FAR END If the line is open circuited at far end, the input impedance may be obtained from the general eqn. 8.23. ie. 25 = z| ZA o08h PL Zo sinh PL = * “9 |Z cosh Pl + Zp sinh Pl 18.23 @)) Dividing numerator and denominator by Ze, we get cosh PI + 2 sinh PL 2k) > * Facosh Pl + sinh PL {8.23 (6)] When the line is open circuited, no current flows, and hence Zp is infinite, then the term bb 4 ZR ~ =z = 7, | cohPl+0 iTtass gis Zs a eee Zs = Zoc = Z coth Pl TRANSMISSION LINES AND MATCHING NETWORKS. 617 When the line is large or the value of, i " < : ren tosses then PY — I and hence Propagation coefficient P is large as in case of line with high 2oc = Z Now the voltage and current along the open-circuited line can be had from eqn. 8.26 (a, b) cosh P(1-x) + 2 gi . ve vg | a ay ae) J cosh P (1) + sinh P(1) Vs cosh P (T=, vy = Ys cosh P(1~x) _ Vs sinh P(I—x sin (Pl) and = Se cost FI [8.28 (a, b)] 8.10. VOLTAGE AND CURRENT WHEN LINE IS SHORT CIRCUITED AT FAR END ‘When the line is short circuited at far end, current flows through the load impedance Zp i.e. Ze = in this case and hence from general eqn. 8.23. cosh Pl + z sinh Pl Zs = Zh Z 5. cosh Pl + sinh Pl Ze = % = Z| wsinh Pl Zsc = Zs = Z [zest a] or | Zsc = Z tanh Pl ++-(8.29) When the line is large, or the losses are high, the value of P] —> © and hence. Zc = Lh The voltage and current of a short circuited line is obtained by putting Ze = 0 in eqn. 8.26 (a, b) V = Vp Sime Um 5° sinh PL --8.30 (a)] a ree 42903 8.11, INPUTIMPEDANCE OF A LOSSLESS R-F. TRANSMISSION LINE OR LOW LOSS TRANSMISSION LINE ; Fora lossless transmission line since, there is no loss (or very low loss). R R=0,G=O0anda= 57 = attenuation const. = 0 Hence, the propagation constant is given as P=at+jp= JB for lossless line as & = 0 (831) Putting this condition in the input impedance eqn. 8.23, we have, = [a (iB) 1+ % ance Z cosh (7B)! + Ze sinh(jB)I But cosh B/ = cosBl; and sinhjBI = jsinB! _ 7, | Ze cos Bl + jZ sin BI 76 a[% cos BI + jZe sinBL wef 8.32 (a)]} 618 ELECTROMAGNETIC FIELDS AND WAVES ‘This input impedance of the lossless transmission line can also be written into an alternative form by dividing numerator and denominator by cos B /. Zr + jZ tanBl Z=2| See is = Zp [ Z + jZe tanBl {8.32 (b)] where Zp = characteristic impedance of line. | Zg = Terminated load impedance. B= an = Phase constant. = length of the line, Zs = Input impedance or sending end impedance of the line. 8.12, INFINITE TRANSMISSION LINE Ifa line of infinite length is considered, then all the power 5 fed into it will be absorbed (Fig. 8.8). The reason being, as we ee $e none nee move away from the input terminals towards load, thecurrentand y,_. 2 voltage will decrease along the line and become zero at an infinite distance. Because the voltage drops across the inductance and the ” current leaks through the capacitance. By considering this Fig. 8.8. An infinite transmission line. hypothetical line of infinite length an important terminal condition is found, At x=0,V=Vs Then from eqn. 8.8 Vs = Ae Po + Beth? or Vs=A+B Further ifx — o°, V — 0, then again from eqn. 8.8, -Po O=Ae?™ + Bet?" = Ae + Bem 0=Ax0+4Be"=0+Be" or Sincee™ # 0,hence Bmustbe zero |B = | Putting eqn. 8.34 into eqns. 8.8 and eqn. 8.28, we have Vs=A and thus VeAe™+0-07™ or | V= Vs e7P (8.35) Also, the eqns. of current for an infinite length of transmission line can be obtained by differentiating eqn. 8.35 w.r.t x. wv -Pr av - o =-Pise oY py ert & ° aoe from eqn. 83 or 1(R+jol)=P- Ve" - ei (R+ jot) = PYs oe _ N(R+J@L)(G+joC) pe Of '= (RF jac) ° = TRE JOL) Ve from eqn. 86 GE+joc pr _ Vs ev or R+joLl Ye Z (8.36) TRANSMISSION LINES AND MATCHING NETWorKs 619 here 7 Jar ” 4 G+joL (837) The eqns. “= and B36 Bive the voltage and current equation at any point of the infinite line. From eqn. 8.36 it is inferred that current entering at the sending end of the transmission line is Vs _ pri J tnd hence Zo indicates the input impedance presented by the infinite line, Thus input impedance ofan infinite line is called characteristic or Surge impedance of the line or in otherwords characteristic impedance is the impedance looking into an infinite length ofthe line. Alternatively, the r-tio ofthe voltage applied to the current flowing will give the input impedance of an infinite line. This input impedance as called as characteristic impedance of the line and is represented by Zp. Vs e7* Vs eo 4 Characteristic Impedance = ¥ Charactaristic Impedance = Zp . 8.13. AN INFINITE LINE IS EQUIVALENT TO A FINITE LINE TERMINATED IN ITS CHARACTERISTIC IMPEDANCE When a finite length of line is joined with a similar kind of infinite line, their total input impedance isthe same as that of infinite line itself, because they together make one infinite line. However, the infinite line alone presents an impedance Zp at its input PQ , because the input impedance of an infinite line is Z.Itis, therefore, concluded that a finite line has an input impedance Zp when it is terminated in Zp or @ finite line terminated by its Zp behaves as an infinite line. This is illustrated in Fig. 8.9 and is proved mathematically too. be FINITE TRAN. UN PFINITE TRAN LINE>j=—INFINTE TRAM. UNE he nannnn= wabr INPUT 129 IMPEDANCE [Zo] IMPEDANCE i —- fea a -}—2—+| (a) (b) Finite line terminated by 2 is equivalent to an infinite line. Fig. 8.9. Equivalence of an infinite line. Leta finite line of length J is terminated by its characteristic impedance Zp and is having volta;- and current as Vz and Ip at terminating end. Zy = Ve/Ie 8,37 (a)] Putting x = 1,V = Ve and J = Jpin general eqn. 8.20, we have Vr = Vs cosh PI ~ Is Z sinh PI and Ig = Is cosh Pl — (Vs/Zp) sinh PI , Vr _ Vscosh Pl ~ Is Zp sinh PI Now by divinding, we get Tk V, I,cosh Pl — % sinh PL +-(8.38) Is cosh Pl -Z sinh Pl Ve or Z by eqn.8:37(0) 620 ELECTROMAGNETIC FIELDS AND WAVES, Vs or Vy cosh PU = Za sh P= Z| cosh #1 — 3s] or Vs (cosh PI + sinh Pl) = Z Js [cosh Pl + sinh PL] or B= % = Input Impedance of the line (8.39) s Vi Hence from eqns. 8.37 and 8.39 Zs = z = - =h or Zs=h -(8.40) Thus the input impedance of a finite line terminated in its characteristic impedance, is the characteristic impedance of the line. Since the input impedance of an infinite line is the characteristic impedance of the line. Hence, a finite transmission line terminated in its characteristic impedance Zo , is equivalent to an infinite line because both lines will have an input impedance Zo, 8.14. CHARACTERISTIC IMPEDANCE (Zo) ‘Any circuit having series and shunt impedances, must have an input impedance. For transmission line, this input impedance depends on many factors e.g. type of line, its length and termination at load end or receiving end. In order to simplify description and calculation, the input impedance under certain standard, simple and easily reproducible condition is taken as the reference and is known as characteristic impedance of that line. The characteristic impedance (Zo ) of a transmission line is defined. (i) “As the steady state vector ratio of the voltage to the current at the input of an infinite line”’ or (ii) “As the impedance measured at the input of this line when its length is infinite or Impedance looking into an infinite length of the line”’. Characteristic impedance is also called as surge impedance and its unit is ohm. The value of characteristic impedance can be obtained by taking the initial eqn. 8.3 of transmission line i.e. av dx However the voltage and current eqns. at any puint of an infinite transmission line are given by eqns. 8.35 and 8.36. 1(R+joL) (8.3) V=Vse* and bet - Z, bY ean (834) (8.36) d Px Px ; ~GWse™ =e" (Rt jl) + Vs Pe = Is e*(R+joL) Vs_ R+jol_ R+joL Is P UR+joaL) (G+jaL) By definition, input impedance, ( Zs ) of an infinite line gives the characteristic impedance Z i. a Ms -VE+ioL Is G+joc or w=(8.14) J Ftiet _.f[Z G+joc” VY TRANSMISSION LINES AND MATCHING NETWORKS 621 This is the equation of characteristic impedance, in which Z is series impedance per section and Y shunt admittance per section. It shows that characteristic impedance is a complex quantity very frequently specially in line communication i.e. telephony. As already shown in previous article, that characteristic impedance of a line can be measured at its input by terminating the line at far end with an impedance equal to characteristic impedance Zp , immaterial what length the line has. Characteristic impedance neither depends upon the length of the line nor on character of the terminating load as seen in Eqn. 8.14. However, it is determined only by the characteristic of the line per unit length. ‘The characteristic impedance is found, physically by the size and spacing of the conductors and by the dielectric constant of the insulator employed between the conductors. As will be seen, the characteristic impedance of two basic types of transmission lines are given by formulae below, 2 = 718 topioS oh Tq (ogio7, ohm (for parallel wire lines) _...[8.41 (a)] and B= AF bos? ohm (for coaxial cable) [8.41 (b)] where k = dielectric constant of the insulation. D = Inner diameter of outer conductor of the coaxial cable. d = diameter of the inner conductor. S = centre to centre spacing between two conductors, and r = radius of the either conductor. It has been assumed that conductors used in construction of transmission lines whether open wire or coaxial (or coax for bravity) cable, are made of copper. For air dielectric parallel wire lines, the formula 8.41 (a) will reduces to Zy = 276 logio 8 ohm as Vk = V1 = 1 for air [8.41 (©)] Eqns. 8.41 further confirm that characteristic impedance depends only on the geometrical configuration of the conductor. 8.15. CHARACTERISTIC IMPEDANCE AT RADIO FREQUENCIES At radio frequencies, when the frequency is so high that resistive component of the equivalent circuit becomes negligible ic.j@ L >> R and jo C 2 G. Thus equation 8.14 becomes Rejok _.fjoL L Royee 7 Njoo 2 ohm (8.42) where L = Inductance per unit length, measured in henries/metres. C = Capacitance per unit length, measured in Farad/metre. Eqn. 8.42 shows that characteristic impedance at radio frequencies is resistive and is measured in ohm. It is purely resistive because there is no reactive term in eqn. 8.36 of characteristic impedance. It may, then, also be represented as Ro. 8.16. PROPAGATION CONSTANT (P) Itis called propagation coefficient or propagation constant because it reveals the nature in which the waves are propagated along the line ie. the manners in which the voltage V and current I vary with distance x, as are clear from the transmission line general equations. V = Vs cosh Px - Is Zp sinh Px [8.20 (a)} 622 ELECTROMAGNETIC FIELDS AND WAVES and Is cosh Px ~ © si S cosh Px = 7, sinh Px {8.20 (b)] where P = Propagation constant P= V(R+ja@L)(G+jaC) perunit length -~(8.10) or P=V oo P=a+iB (8-10, by The propagation constant of a uniform transmission line per unit length is defined as the natural logarithm of the vector ratio of the steady state voltages or currents entering and leaving the structure ie. Vs P = loge & or P= loge ms 18.43 (a, b)] Since propagation constant is a ratio of voltages or currents and hence has no unit i.e. dimmensionless. However propagation constant is expressed in per unit length i.e. per km or per metre, When the line is lossless propagation constant is purely imaginary and is directly proportional to the frequency as shown below. However, propagation constant for lines with losses, is a complex quantity and in general, is a complicated function of frequency. When propagation constant is multiplied by the length of transmission line, then it is termed as electrical length of the line. 8.17. ATTENUATION CONSTANT AND PHASE CONSTANT Propagation constant is a complex quantity having real and imaginary parts as P=a+jB -~(8.10) P=V(R4J0L)(G+ oC) = pou (1+ 5g ac{t+; jo 1 1 =VF@Le V (+727) V(+752) ~10V2(1+782] (700) SincewL >> R and oC >> G 2f 2. or < VE + TVS [8.43 @)] and B=0 VIC {8.43 (€)) Also Vp = 1 “Te 0 Vp = Te {8.43 (1 le TRANSMISSION LINES AND MATCHING NETWORKS 623 The real art is known as Attenuation constant and is expressed, in Neper per km. It represents she attenuation or reduction of voltage and current along the line. The word Neper is derived from the Napier which is distorted form. ss ia eee par is aid Phase constant or coefficient or wave constant. It is called so ra vartarion i wave suffers a phase change of B radians. It is expressed in radian per km. The variation in phase positionof voltage and current along the line is determined by it In fact, and B both are function of frequency and hence also sometimes referred to as attenuation function and phase function. Although a. and B are expressed in Neper and radian but they can be connected into more practical units like decibels (db) and dey i tiplying 8.686 ena? This is Because (db) and degrees respectively by multiplying 1 neper = 8.686 db [8.43 (g)] and 1 radian = 57.3 degree (8.43 (h)] If propagation constant is multiplied by the length of the transmission line, then it is termed as total attenuation of the line or line attenuation simply. ___Asillustrated in Fig. 8.10, the locus of the current vector is a log spiral when attenuation exists and in the absence of attenuation it is a circle. This is because propagation constant being a complex quantity P = a + jB,so bs Ik Thus as J; enters the line (Fig. 8.11), and reaches a unit distance apart at point BB’, it would be attenuated in magnitude by an amount a and phase will be retarded by an amount B radians per unit length. In fact, due to this attenuation, both incident and reflected waves suffer a change in amplitude of e* per unit length of the line. =P a ettiB at. oid Is an gk ois 0 Y 4 vs© vs 2 v 7 * 3 8 betaned Fig, 8.10. Locus of the current vector. Fig. 8.11. 8.18. DERIVATION OF EXPRESSIONS FOR a AND B IN TERMS OF PRIMARY CONSTANTS (ie. R, L, G, C) ; The Propagation constant ( P ), Attenuation constant o, Phase constant (B)) and characteristic impedance ( Z») are called as secondary coefficients or secondary constants of a transmission line and all these four constants have tendency to vary with frequency. All these secondary constants can be calculated if the primary constants of a line are known, as shown, for that frequency. Since p=V(R+jOL) (G+jac) =a4+ jp 8.44) (Prove it as in Eqn. 8.6, from first principle). = of + (JB) + 2408 = (R + J@L) (G+ joc) ... by squaring both sides = RG +j@CR+jOLG—w LC... onmultiplying On arranging, we have of - BP + 2jaB = RG - w LC + jw (LG + RC) 624 ELECTROMAGNETIC FILEDS AND WAVEg Equating real and imaginary parts, a — 6° = (RG - w? LC) +{8.45 (a) 2aB = w(LG + RC) [8.45 (b)] ‘iss |P|=Vea Be +» from Eqn, 8,38 a =WVR +0 L)(F +a C) Squaring both sides, we get +P sVR +a Ll) (+a C) (8.46) Now adding Eqns. 8.45 (a) and 8.46 20? = (RG - w LC) +V(R 40° L) (G+ aC) or a=t+\ $ U(RG - @? Ley] + VOR +@ L)(G rae) | ~-[8.47 (a)] Subtracting, we have - 28? = (RG - 0 LC) -V(R +0) (@-a@ C) or B= -3[ (RG - ot 1c) -VR FEL) (Fre) | B= 3[- (RG - 0 Le) +VR +a LT) (F +a F) | or B=+ 3 [wt ee - RG) + VR TLV) | Eqn. 8.47, gives the values of attenuation constant o. and phase constant B in terms of primary constants, i.e. R, L, G and C. In practice, approximations can made in order to simply if the expressions for all the secondary constants under particular conditions. For example, in paper insulated telephone cables, the conductors have relatively small diameter and are close together. The inductance and conductance are small and hence at audio frequency [8.47 (b)] R>>@L and MC>>G Under this condition P can be written as P=V(R+j@L)(G+joC) =V(joc)xR a+ jp = oCRx 1 =VaCR 245° = VOCR [cos 45* + jsin 45"] = V@CR cos45° + jV@CR sin45° = VoRC x tier xe _4fock , .focr. a+jp= 7 td 2 Equating real and imaginary terms, we have| |o:| = |B] = - (8.48 (@)) Characteristic impedance Zp in this case is Zz. R+jok _ R js Bia) Bg pe %=NY = No+iac ~ Vjwe*j = Ve ae = Vae* TRANSMISSION LINES AND MATCHING NETWORKS 625 2 ac 2-4 8.19. ars UNITS OF A TRANSMISSION LINE esides, primary constants (R, L, G, C) and secondary constants ( P, c., B , Zo )» there exist thee re pee as cite line, e.g. wavelength, velocity of propagation or phase . Thess veloc am ¢ can be calculated, if at the frequency of operation and the value of (i) Wavelength (2). It is defined as the distance in which the phase change of 27 radians is effected by a wave travelling along the line or. The wave length of a cycle is the distance between two successive positive or negative peaks as illustrated in [8.48 (b)] Fig. 8.12. It is denoted by the Greek letter (A) and its -—— unit is metre. ae) La __.| Hence by definition pA=2n Fig. 8.12. Defining wavelength of a wave, an or A= (8.42) 8 Since, as the distance increases from O to D along the line, the value of Bx must change by 2m i.e, at the same time that x changes by 2. (ii) Phase Velocity or velocity of propagation (V, ). It is defined as the velocity with which a signal of single frequency propagates along the line at a particular frequency (f ). It is measured in km. per second. It is the velocity of propagation along the line based on observations of phase change along the line and hence often also called as ‘*phase velocity (Vp )"". The change of 2m radians in phase angle represents one complete cycle in time ¢ and occurs in a distance of one wavelength ( .), 50 LeVp t= Vpr4 f or : [Meek F] where f = frequency {8.48 (@)] 2n o or w= Bla B Inf=o or Vp = B km/sec. 18.48 (b)] Velocity of propagation of wave can never exceed the velocity of light. However in RF. lines having air dielectric, the Vp approaches the velocity of lighte.g. 3 x 10° m/s or 3 x 10° km/sec. Git) Group velocity (Vg )- Ifthe transmission line or transmission medium s such that different frequencies travel with different velocities,then the line or the medium is said to be dispersive. In that eee velocity (V,). The group velocity is usually cases signals are propagated with a velocity less than the phase velocity. The inter-relation between-group velocity and phase velocity can be shown tobe oof 8.45 (a, BD] where 2 and q; are two close angular frequencies being transmitted and 8 and Bj the corresponding phase constants, The dw»/d B is measured at the centre frequency. 626 ELECTROMAGNETIC FILEDS AND Waves s 8.20. CALCULATIONS OF PRIMARY AND SECONDARY CONSTANT! Can be done by simplifying the results already derived. For, if Zo and P are known, primary constants R, L, G, C can be calculated and vice versa is also possible. [zZ_.[ R+iet , m= V2 -V ete (8.14) P=V(R+J0L)( 6 +j0C) (8.10) %xP=R+joL by multiplying [8.50 (@) and Pugsjoc .» by dividing [8.50 (b)} Z ‘Thus from Eqn. 8.50, by equating real and imaginary parts, R, @ L, G, @ C can be calculated and if frequency () or angular frequency, (@ ) is known, then L ,.and C are obtained. For calculation of secondary constants the Eqn. 8.14 are again used and hence if R, L, G, Care known P and Zo can be calculated. By equating real and imaginary parts in P, theo. and B can be obtained. 8.21. CALCULATION OF SENDING END AND RECEIVING END PARAMETERS For this consider the figure shown in Fig. 8.13. ° 5 “vq Ys Zs" 2 SENDING = END aro RECEIVING END | (a) Transmission line with Zg and Vg. (b) Equivalent of sending end circuit terminated in Z). Fig. 8.13. ‘When a uniform transmission line is correctly terminated and open circuit voltage of the generator and its internal impedance is known, then the figure shown in Fig. 8.13 is helpful in calculating sending end parameters. In above figure the uniform transmission line is replaced by equivalent lumped impedance Zs. Once the sending end voltage Vs is calculated from eqn. Vevse™ the voltage and current at any point on the line can be calculated. From the equivalent Circuit V, [Us| = Zaz [8.51 (@)] [Vs| = [is Zs] [8.51 0) Average power entering thelineis | Ps = |Is/’ Rs =| Vs| | Is| Ro [8.510] where Ro is the resistive component of Z) = Zs. 8.22. OPEN AND SHORT CIRCUITED LINES Since the impedance at any point of the transmission line is the ratio ofthe voltage to the curremt and thus the impedance is also dependent on the type of load at the far end and the distance from it. 10 other words, in any transmission line the terminating load establishes the current and voltage relation: The various methods by which the voltage and current may be distributed along a transmission line Ca! be simplified by considering three-important cases of the terminating load TRANSMISSION LINES AND MATCHING NETWORKS 627 (@) When the terminating end is open circuited (6) When the terminating end is short circuited. (©) When the terminating load is equal to the characteristic impedance. By open circuited we mean that far end of the transmission line (. at load end) is open while in case of short circuited this end is shorted by a metallic strip. Both open circuited'and short circuited T/L (transmission line) of suitable length can be arranged to act as series resonant or parallel resonant or any desired value of reactance i.e. inductance and capacitance. Athigher frequencies the length of such transmission line is small and hence are used as reactances. The short length of 7/L, then called as “‘Stubs’’. Short circuited stubs at high frequencies are preferred rather than open circuited stubs due toradiation losses at the ends. ‘As seen in Eqns. 8.11 and 8.12, that there are two waves along the line which are travelling in opposite directions between sending and receiving ends. They are called as incident waves (travelling from sending to receiving end) and reflected waves (travelling from receiving end to sending end). Because of this there will be interference between the two waves and reinforcement and cancellation will take place. The points where the two waves are in phase, will add and produce voltage maxima, while the points where they are in opposite phase, will cancel and will produce voltage minima. The points of voltage maxima and voltage minima are also called as antinodes and nodes respectively. Since in case of open and short circuited lines, the complete reflections occur and thus there is complete cancellation or the voltage minimum is zero. In other words positions of voltage maxima (antinodes) and voltage minima (nodes) remain motionless and so standing wave is said to exist on the line. (a) Open circuited line. Since no currentcan flow in an open circuit, although voltage can exist between two wires. Therefore, when the line is open circuited, there exists a maximum voltage and minimum (rather zero) current. This implies that impedance of open end is infinite. At a point 1/4 from the load end the incident wave is 90° earlier while the reflected wave is 90° later than what they are at the Joad end. Thus there is 180° phase difference between incident and reflected waves, resulting in voltage ‘minimum at the 1/4 point. The standing wave pattern is, by similar reasoning repeated every 2/2 length ___ (Short Circuited Line) Open Circuited Line. VOLTAGE CURRENT CURRENT VOLTAGE | OAS KO vourase— NODES (a) Lossless line. | VOLTAGE L aaa RO We (2) Lossy tne. b) Lossy line. Fig. 8.14 isribuionof wltage and currentalong open Fig. 8.15, Disibuion opcoage and curren along shot circuited lossless and lossy T/L circuited lossless and lossy lines. 628 + maxima and mifima are spaced 2/2 apart. The current maxima occur at points where the voltages minima are and vice-versa as shown in Fig. 8.14. At high frequencies lossless lines, the voltage maxima values are equal but in lossy lines it go on decreasing due to attenuation. (b) Short circuited lines. When the load end is short circuited, then there exists no voltage difference. However, currrent flows. Thus at short circuited termination, the current is maximum, vol is zero and so the impedance (i.e. zero). Therefore, unlike open circuited termination, the standing wave have minimum voltage or node at short circuited end the voltage minima are spaced at every 2/2 apart from the load end. The voltage and current distribution along short circuited lossless and lossy lines are shown in Fig 8.15. In lossy line, the voltage and current gets attenuated as they travel towards load end. ELECTROMAGNETIC FILEDS AND Waves (© Line terminated with characteristic impedance Zp. The transmission lines used in Practice, are not infinite length and it is required to deliver power to load from the generating or source end. Ifthe terminating impedance is equal to the characteristic impedance of the line, then the line behaves as an infinite line and the input impedance of this line will also be equal to Zo. This line is said to be correctly terminated and all the power sent from source is absorbed in the load, and hence there is no power loss due to reflection. 8.23. INPUT IMPEDANCE OF OPEN AND SHORT CIRCUITED LINES Input impedance of an open circuited and short circuited lines is measured at the input terminals ofa finite length of transmission line when its far end is open or shorted respectively. They are respectively denoted by Zoc and Zsc . Let the line length be /, load end voltage Vg and current /p, then by eqn. 8.22 Ve = Vs cosh Pl Is Zo sinh PI 8.22 @)] Vs I = Is cosh Pl - Z sinh Pl l8.22.(0)] When line is open circuited, then Ip = 0 O = Is cosh Pl ~ # sinh Pl from 8.22 (6) . Vs cosh PL _ F or Zoc = 75 = Ty Sarpy = Zo colh Pl 1852001 When the line is short circuited, then Ve = 0 O = Vs cosh Pl ~ Is Z sinh PI from 8.22 (a) Vs _ Zsinh Pl ate, SAT oo, ick or Bsc = 75 = “Can pr = 2 tanh PL on or or On multiplying eqn. 8.52, we get Zoc + Zsc = Z coth Pl + Z tanh Pl = Z ir Z = VZoc + Zc --(8.53) Zsc _ % tanh Pl _ 2 Zoc ~ Z cath Pl = nh PT On dividing, we have tanh PL = A 2 Zac (8.54) TRANSMISSION LINES AND MATCHING NETWORKS ad Eqns. 8.53 and 8.52 are of great practi Practical impedance Zo and propagation const, P a R,L,G, C, can also be calculated as already explained. 8.24, OPEN AND SHORT CIRCUITED LOSSLESS TRANSMISSION LINES For a lossless line of length J, the vol “For a age end eqns. can be obtained from the general transmission line eqn. 8.20 by putting x = LV = Ve andl gala ° neem alues as just by measuring Zoc and Zsccharacteristic can be calculated and by knowing Zp and P, primary constants Va = Vs cosh Pl - Z Js sinh PI 2 cosh Pl = cos BI = Vs In = Is cosh Pl — % sinh PI sinh jB / = jsin BL For lossless = 0, Zs P=jPasP=a+jp Va = Vs cosh j Bl —Js Zp sinh j B= Vs cos Bl -j Is Zo sin BL {8.55 (a)] and Ik = Is cos Bl ~ j z sin Bl -uf8.55 (b)] For open circuited line Ip = 0 (Zp = 2) Vs Vs 0 = Is cos Bl 7 8m Bs Bur Zoe = TE Vs Vs _ ZH cosBl j Js cos BI = j S sin Bl Ms Ach i Is cos Bi iz, sin Bi or ie 7 jain 5 Zoc = — jcot Bl -~-[8.56 (a)] Similarly, for short circuited T/L Va = O (Zr = 0) . Vs _ , Z sin Bl _ O = Vs c0s Bl jls Zo sin BL or 7 = j “BT = Zoe or Zsc = j & tan Bl ~-(8.56 (b)} Eqn. 8.56 shows that input impedance of af open and short circuited lossless transmisson line is a pure reactance as viewed from the sending end. The value of reactance can be changed by changing the electrical length B I stubs. The reactance of eqn. 8.56 (a) is inductive of the value of cot B | is negative and capactive when cot Bl is positive. Similarly in eqn. 8.56 (b) input impedance is inductive when tan | is Positive and capacitive when the tan B 1 is negative. The variation of input impedance Zoc and Zscas a function of B /is shown in Fig. 8.16 in which dotted lines represent the variations of Zoc with B 1 or / and full line the variation of Zsc as a function of BI or £ Above horizontal line the value of +00 Zoc and Zc, will have positive reactance ie. inductive while below the horizontal line, the value of Zoc and Zsc will have negative reactance i.e. 2o¢ capacitive. 2se The figure indicates that if the length is between 0 to A/4 so that angle B/ between 0° to 90°, the open circuited line will be capacitive (negative reactance of eqn. 8.56 (a)) and short -ol Circuited line will be inductive [positive reactance —Ve=-REACTANCE=+ Ve —= UNE LENGTH Fig. 8.16. Variation of Zoc and Zsc with Bl or 630 a coititiice PAA he, eqn. 8.56 (b)]. When the length is between 4/4 to 1/2 B/ lies between 90" to 180° and both tangent and cotangent are negative. Hence open circuited line becomes inductive and short circuit line capacitive. This behaviour interchanges after successive increase of length of 4/4. Thus such section of T/L is utilized ag reactive elements (capacitance or inductance) at higher frequencies. Open and short circuited lines also behave like resonant circuit. This happens when length of line is integral multiple of 1/4. ie. BI = nA/4, | then tan By = © and cot Bl = 0, 2n dh tn For zoca jon [SEG] =-s om 3 0 Zoc = 0 [8.57 (a)] Tt means open circuited lossless line presents zero impedance at source end. (2m Similarly Zsc = j tan (ie : ‘ =jtunt/2=0 Zc = 18.57 (b)] This means a quarter wave short circuit line presents an infinite impedance at input terminals, just like a parallel resonant (LC) circuit and a 44 open circuit line present zero impedance at input terminals just like a series resonant LC circuit. This behaviour is interchanged at each successive M4 increase of the line length. Hence a transmission line act as resonant circuit and is used in many applications at high frequencies in antenna design and other radio circuitory. 8.25. TRANSMISSION LINE MEASUREMENTS AND DETERMINATION OF PRIMARY AND SECONDARY CONSTANTS BY BRIDGE METHOD It is evident from the preceding articles that primary costants of a transmission line (i.e. R, L, G, and ©) and secondary constants (P, &, B and Zp )can be determined, ifthe input impedance of the line is measured under the following two conditions : (i) The far end open circuited, (ii) The far end short circuited. The attenuation coefficient ot, Phase coefficient B along with R, L, G, and C are computed from the values obtained from Zp and P. The input impedance of a transmission line is measured under above conditions with the help of ana.c. bridge known as WIEN Bridge (Fig. 8.17). In this bridge, the arm PS is connected to the transmission Tine to be measured. In fourth arm Fig. 8.17. WIEN Bridge method for RS a variable resistance R and a variable capacitor impedance measurements. Care connected. The first arm PQ and second arm QR contain resistances R, and Rp which are usually Pace auatin calue boon as waive ratio arm. A variable capacitor C shown by dotted lines in the itn PS. = ° n the line impedance is negative. In order to get the impedance Zs across fourth R and Cinthirdarm RS are so adjusted that null condition (ie. no current condition) is obtained TRANSMISSION LINES AND MATCHING NETWORKS 631 in Galvanometer G. Under this condition, Riz RZ where Zs = Input impedance of the line in arm PS Z = Impedance across arm RS. Let: and Ry are so chosen thats values are equa ie. Ri = Rp of BE = 1 and hence 1=<3 / This indicates that under null condition the impedance of third arm Z_ is equal to the input impedance of the line. Let us, then calculate the value of Z, the impedance of third arm. 4, daC _1+joCR Ss Zz R or —R__, 1- joer 1+joCR”™ 1-jocR =R@M-joCR) (4+B)(A-B)=A°-B? Rationalizing this, we have Z = Zz 2 1+ (@CRY away = Rk __ _jocr or Z=X4 iY (88) = Tory 1+ (@CRE Equating real and imaginary parts, we get fl __-0ck X= Teter? and Ye Tt (oCRY [8.58 (a, b)] R he ock | ws 2-vPrF = Vota) tical ; . (leaf I owl ; R 1 2 \21= Viyqery * [1+ (ory x{1+(wery] R 18.59 (@)] 1Z1= We (oery [ ‘erat gt St and the phase angle @ is given by gqtan'y If the capacitor C is connected across the line the value of Z is the same as given above, but [8.59 (0)] a HO CRY aay! (-ocr) — ~18590)1 @ = tan’ (+ MCR) 632 ELECTROMAGNETIC FILEDS AND Waves In case the line has negative phase angle the condenser C must be transferred to the fourth arm PS either in series or in parallel depending upon the value of the reactive component of the line impedance, Now let us assume that input impedance measured under far end open circuited be !Zoc| 2 @oc and under far end short circuited be | Zsc| Z sc, then Zo, P and other constants can be determined as follows 8.25.1, Experimental determination of characteristic impedance Zo. After knowing the value Z and hence Zs under open circuited condition and short circuited condition by an Wien A.C. bridge Z is given by % = VZoe x Bsc = VZoe Z Goc * Bsc Z Osc tm =VZoe% tac Z pee we 18.60 (a)} This gives the value of Z . As a measure of check, after calculating Zo as above, the far end is connected with the value of impedance Zy and Zs is computed again. This should be equal to Zo because the input impedance of a line terminated in its characteristic impedance, is the characteristic impedance. 8.25.2. Experimental determination of Propagation Coefficient P. Once the value of ZocZ@oc and Zsc Z Osc are determined from the A.C. bridge measurements, i.e. the propagation coefficients are determined from the formula. --{8.60 (b)] --[8.60 (c)] This shows that the value of P can not be obtained directly unless simplified. For, Let the R.H.S. is converted in to cartesian form, then tanh Pl = Z [cos@ + jsinO]=M+jN (say) «(8.60 (@)] ae ete Applying componendo and dividendo we get (i.e. Numerator + Deno. divided by denominator numerator) or Mae MeN M+jN+l | 2e" 14+M+jn Pei Ma eh 1-M-iN 2eF " 1-M-jN or {8.61 (@)] For changing to polar coordinate, let RHS is written in terms of polar form as ert. n 281 m ZO e728 (say) [8.61 (b)] But loge r 20 = loge r e(@* PAR) ~-[8.62 (a)] loger 20 = loge r+ j 2 ee | 2 Nz loge r+ J (0 + 2H) loge € 18.62 (b)] TRANSMISSION LINES AND MATCHING NETWoRKs 633 is the same for mbiguity arises Iemay be noted that term 2.n x has been introduced, as the vector r e!(®*2*®) all values of n but the logarithms of the vector depends on the value of n. In this case a1 because the line may be more than one wavelength long and this is not directly noticed from the calculations. From the measurements we calculate a certain Phase angle for the vector Pl, but if the line is several wavelengths long the vector will have made several revolutions in addition to the calculated angle and this is taken into account by adding the term? nt. Hence from 8.61 (b), taking log of both sides we have ea py, gil8 + 2nx) 2 PI loge € = loge r + j (0 + 2nm) Since loge € = 1 1 . or P = 5) (2.303 login r + j (0 + 2nm)) ED) Since loge r = 2.303 logio r n=0,1,2,3.... = positive integers Alternatively, the value P and hence the value of « and are determined from eqn. 8.60 (c). tanh Pl = M + jN or tanh (a + fB) 1 = M+ jN (8.64) Similarly, the complex conjugate is tanh (a —jB) l= M-jN (8.65) Now, tanh (2 af) = tanh [(a + jB)/ + (a - JB) 1] _ _tanh (ot+B)J+tanh(—jB)1___(M+jN)+(M=jN) ~ T+ tanh (a +jB)¢xtanh (a—jB)1~ 1+(M+jN)x(M-jN) h2al = 2M _ 2M nn Ot TP) aa 1 a 2M or a = 57 tanh [hel (8.66) Similarly, tanh (j2B2) = tanh [(o + JB) !-(a-jB)E] tanh (a + /B)!— tanh (a-jB)! Since tanh B=jtanB = [= tanh (a+ 7B) 1x tanh (a — jB)1 _(M + jN) = (M4 ~jN)_ “1 - (M+ jN) x (M -5N) j2N 9 By ofa 2N jm pie pg 126 ww! (a 2N or Tom oN (8.67) “Thus, the value of a may be calculated from eqn. 8.66 and the value of f may be determined from the eqn. 8.67. Let 1 2N “1 2N — — Ne 2 BI = tant B=7, a (=e) °r Br = tant (a) or 2B! = 0 radians (say) or 2BL= oxen degree --{8.68 (a)] 834 ELECTROMAGNETIC FILEDS AND WAVES In general, 2pr={2x2y 8 Br | igo | +" rad Oxn) nm fi or = {(Oxey 22] ayunitiengtr | B t(Ser] ea} iM 2 | [8.68 (b)] Lot) In order to know the vaiue of n, the approximate value of B must be known. 8.25.3, Experimental determination of 0, , vp « It is shown that P, the propagation coeffi. Cient is calculated from the known values of Zoc and Zsc determined from bridge measurements. The real Part of P gives the attenuation constant 0. and imaginary part of P, gives the phase constant B, as P=at+jB a+ J P= [loge r+ j (0 + 207)} .. from eqn. 8.63 or te = Le tog 2 7 logio r Neper/unit length --[8.69 (a)] and + (@+2nm) Radians/unit length ---[8.69 (b)] The value of B is known, only when the value of n is known. For, the approximate value of vp or B must be known. Since vp = O/B If the velocity of propagation is given in the problem, calculate the approximate value of B from B= w/v --(8.70) After obtaining the approximate value of B, the corresponding value of n is calculated from eqn. 8.69 (b) by assigning the value n = 0, 1, 2, 3, 4, Rounding of this value of n will give the desired value of n. Now if this value of n is substituted in eqn. 8.69 (b), the correct value of Bis determined. Once the correct value of B is known the V, is calculated from the relation \ = O/B. eqn. 8.26. DECIBEL AND NEPER The units of attenuation are decibel and neper. As has been seen that in passing along a terminated line a signal is attenuated in amplitude and retarded in phase. The extent to which the attenuation takes place depends on the magnitude of o: the attenuation coefficient and B the wavelength coefficient. The B gives the phase shift in radians per unit length. In order to find the attenuation represented by ot, let us first consider the units of attenuation. > Attenuation was initially defined in terms of the attenuation cable, as known historically. A circuit was then said to have an atten cable{M.S.C). The main objection to this unit was the fact that the attenuation of the standard cable chanees with the frequency. With the introduction of systems operating over different frequency ranges it became necessary to define a unit of attenuation, which does not depends on frequency. The unit so selected was the Bel but in practice a submultiple the Decibel is, usually used. The telephone industry proposed and popularised the unit based on logarithms to the base 10, naming the unit for Alexander Graham Bel. For, consider a four terminal network having input and output currents Js and dk respectively. Produced by a standard reference uation of so many miles of standard TRANSMISSIUN LINES ANU MATCHING NETWORKS 638 Further, if Ps represents the power supplied to a Frenuaion ofthe network ais given’ey network and Px is the output power from a network, then Number of Bells = dar = logia a Bells (8.71) R The bell has seve defined as the logarithms of a Power ratio, However, the unit one-tenth as large is more convenient and this smaller unit is called the decibels abbreviated as “db” and defined as © = 10 logio e decibel or db 8.72) If the value of impedance is same for both the eases of powers (le. Ps and Pr) then, nc? -eememae os oe @ = 10 logio que = 10 logo (#] ue Zp = Ze ol8.73.(@)] or @ = 10 x 2 logio (Z) = 20 logio (2), {8.73 (6)] on © = 20 logio (¥ (8.74) Here the subscript Sis for input and R is for output quantities. The decibel is one-tenth of Bell and the former is preferred because the latter is inconveniently large. If Ps > Pr the power ratiois positive and if Ps < Pa the power ratio is negative. For example, Let Ps = 50 W Pr=10W Then a = 10 logio 2 decibels = 10 logio 5 db = 10 x 0.6990 db = 6.99 a=+7db In case of vice versa i.e. if Ps = 10 W and Pr = 50 W, we get a = 10 logo (33) decibels = 10 logio 10 — 10 logio 50 db 10 x 1.0000 - 10 x 1.6990 db = 16 - 16.99 = - 6.99 a=-7 db Since decibels is a power ratio, therefore it can not be itself used as to express an absolute level of power. Power level may however be expressed in terms of some arbitrary reference level, ie. say 1 mW in a resistive load of 600 ohms, the one of the common value. For this datum a power of 3.98 mW in 600 ohms resistance circuit could be expressed as m © = 10 logio 38 decibels = 10 logio 3.98 db = 10 x 0.5999 db = 5.999 db a= + 6db with reference to 1 mW. ‘Sometimes the reference level is indicated in abbreviated form, like db mW i.e. ‘‘decibel relative to 1 milliwatts,” or db wu V or dbV decibels relative to 1 mico-volt or 1 volt respectively. Further, in describing absolute values of current, voltage or power, the reference level must be stated otherwise it is meaningless. For example “‘a power of 20 db” is meaningless but ‘‘a power of 20 db relative to 1 W"” means a power of 10 Watts. For Ps = Ps Ps a = 20logio pdb or 20 = 20 logig 7 636 ELECTROMAGNETIC FILEDS AND Waves 2 or Jogi = 7 a 21 or = = Antilogio 1.0000 = 0.1000 x 10? = 10 Watts. Alternatively, Neper a another logarithmic unit originally introduced to express the tention of current along a transmission line using Natural or Neperian (and hence the a pes) Nog rather than common logarithm. The unit is based on a natural logarithms of current ra 0. network w ich has an input current /s and an output current Ja, then their difference N’ in Nepers has an ion of Vs Re (8.78) N = loge £ Nepers = loge Ifthe current passes through the same impedances, the attenuation may also be expressed in terms of power ratio i.e. input power to output power or and hence trom eqn. 8.75 N= Ps | N= 5 loge p, Nepers uf8.76 (@)] = Vs N= lobe | Z| £8.76 63] [8.76 (c)] or 8.1) ‘Thus two voltages or currents, differ by one Neper when one of them is ¢ times as large as the other. The number of Nepers represents a convenient measure of the power loss or gain of a given network. Losses or gains of successive networks then may be introduced by addition or subtraction of their appropriate N values. Let us now bring out a numerical relationship between Neper and decibels. 2 Attenuation in db = 10 logio (i) Ik Z * (a) db = 10 logio | FE | = 10 x 2 toguo | # | By eqn. 8.77 a = 20 logio {e*} = 20x vx logig € =20.N x logio 2.71828 : ro 1 } Since e=Limit}1+—=)=14 1,3, 1 o pie 2) erecheph epee [_¢= 271828 TRANSMISSION LINES AND MATCHING NETWORKS 637 Hence (O)us = 20 N x logig 2.7181 = 20 N x 0.4343 Cui = 8.686 N [8.78 (a)] Therefore, [(Attenuation ja = “ ( Attenuation )ay = 8.686 x Attenuation in Neper ++ [8.78 (b)] Thus if N = 1 Neper (Attenuation )gy = 8.686 x 1 Then from eqn. 8.78 (a) we get or Attenuation (a) = 8.686 db ie 1 Neper = 8.686 db su(8.79 (@)] or Attenuat = A Neper wation in db = “NBS or Attenuation in db = 0.1151 Neper ie. 1 db = 0.1151 Neper 18-79 )] Eqn. 8.79 are the relationship between decibels and Nepers of a line per unit length. Decibels and Nepers being logarithmic units are particularly useful in communication equipment as the gain or loss of a system connected in cascade can be obtained simply by addition or subtraction rather than multiplying or division which would be necessary with a linear system of units. Further, since human senses like hearing and vision, percieve changes in logarithmic rather than a linear scale and hence the preference of logarithmic unit. Most of the energy transmitted through electrical netwoks, lines, and filters is converted ultimately to accoustic energy and heard by human ear as sound. The important property of human ear may be stated as “she ear hears logarithmically” or the ear is observed to obey the Weber-Fechner Law. This states that ‘“The change in stimulus necessary to produce a perceptible change in response is proportional to the stimulus already existing’’. The ear, therefore, hears sound intensities ‘ona proportional or logarithmic scale and not on a linear scale. The loudness may, therefore, be expressed as L=K lop S where K = constant of proportionality ; 6 = base of the logarithms ; $= Sound power Now let a sound of original power Ss and loudness Ls is changed to cause in increase of sound power Sz with loudness Le , the change in loudness is, therefore, Si AL = Ln Ls = K logy Sx ~ K logs Ss = K logy 5 (8.80) Let us assign different values to the values of K, 6, the change in power is expressed as follows in Nepers, Bels and Decibels . — 1 Sr (@) IfK = 4 and b = e,then from eqn. 8.80. | AL = 5 log. * Nepers (8.81) Se - AL=1 =e ), b = 10, then 0 logio Sr or AL = logio ‘5° Bels (8.82) (©) If K=10,b=10, then AL = 10 logio a decibels (8.83) 5 Hence the logic behind the logarithmic units e.g. Decibels and Nepers is clear. 638 ELECTROMAGNETIC FILEDS AND Waves 8.27, REFLECTION | ‘When power is applied by the generator at input end to a transmission line of infinite length or tg a finite transmission line terminated by its characteristic impedance all the power is absorbed by the load at far end and there exists no reflected wave, However, when the finite transmission line is terminated by an impedance other than characteristic impedance, then only some of the power is absorbed at the load end and rest power will be reflected back to generator end and thus reflected waves are setup. Thisincorrect termination gives rise to inefficiency and the inefficiency is larger, if the difference between characteristic impedance and load impedance is large. | , Thus reflection of energy occurs whenever there is an impedance irregularity i.e. termination impedance is not equal to characteristic impedance of the line or primary constants of the line are not uniform along the line. Hence the phenomenon of setting up ofa reflected wave at the load due to improper termination or due to impedance irregularity in a line is known as reflection. The reflections occur in all cases of travelling wave where discontinuity of medium is involved. The reflections in transmissions line appear, because the ratio of the electric and magnetic fields on the both sides of the terminal boundary (i.e. sending and receiving ends) must be the same in all conditions. This condition can be satisfied by the addition of a reflected wave to the incident wave, Although reflection is normally undesirable on the transmission line. Reflected wave may appear as echo at sending end provided attenuation is not large. It may further be reflected from the sending end and thus energy is oscillating back and forth on the line until die out in the line losses. Reflection is extreme when the line is open or short circuited and it will be zero when Zz = Zp. Under open circuited condition when the incident wave approaches at the end, the magnetic field collapses because the current is becoming zero there. This collapsing magnetic field produces an electric field which is added to the existing electric field and thus, voltage at the open circuited end is increased. This additional voltage gives rise to a wave which travels back to sending end. Since there is nothing to absorb incident energy at the open end and hence the reflected wave too has the same magnitude and phase what the incident wave has at the termination. Thus voltage doubling action is said to occur with open circuited lines. Since reflected wave is in phase opposition w.r.. incident wave so the resultant of the two waves is zero. Similarly under short circuited condition, electric field collapses on the shorting strip, the charge on the line is reversed at the short circuit and a reflected wave is set up again. At the front of the reflected wave current doubling is said to occur while voltage is zero. 8.27.1, Reflection Coefficient. Reflection coefficient is defined as the ratio of the reflected voltage (or current ) 10 the incident voltage ( or current ) and is usually denoted by K or r orp « Reflection coefficient is a vector quantity having magnitude and direction as well. When dealing with voltage ratio (rms values) itis called voltage reflection coefficient (V.R.C ) and when dealing with current ratio, itis called current reflection coefficient (C.R.C. ). But the current reflection coefficient is negative of voltage reflection coefficient because, reflected current is taken as negative since reflected voltage if considered positive will circulate an opposite current to the previous one. Let V; and V; be the incident and reflected voltages respectively. Then by definition, Ve Te vas K Also i -K [8.84 (a, b)] Now a mathematical formula for relfection coefficient can be derived from the fundamental transmission line eqns. 8.8 and 8.13 reproduced below VaAe™ + Beth A8.8) (Ae — BetPs and em) 6.3) ‘TRANSMISSION LINES AND MATCHING NETWORKS 639 Of these the first term is representing inci ident d term soltecied voltage or current, ig incident wave voltage or current and the secon: Let y be the distance measured from the termi inating load im Zp, then above eqns. an be expressed in terms ofy simply by putting x = ye Oat imPedanee Zn. then Then VeAet? 4 Bey [8.85 (a)] fe az (Act? — Ben?) sof 8.85 (b)] In these equations too, the first term and second terms respectively represent voltages or currents as incident wave must go on increasing and reflected wave decreasing as the y increases. Applying the condition, when y = 0 V = Vp and I = ip Ve=At+B += [8.86 (a)] A-B and IR = from Eqn. 8:85 [8.86 (b)] or A-B=ZIk and A+B=Vp 2A = Vat ZH Ig .....0n adding or A= athe and B= Yan fole [8.87 (a)] from Eqn. 8.85 (a) or on dividing both sides by Iz _ Vr o 2 tae or (8.88) ‘This is eqn. for reflection coeff. and is in complex variable eqn. as Ze is complex. Hence reflection coefficient can easily be calculated if load impedance Zp and characteristic impedance Zo are known. An alternative derivation is also given below and either can be used. Alternative. Let a T/L no. | of characteristic impedance Zp is connected to another T/L no. 2 of characteristic Za, which is correctly terminated. Reflection will occur at the junction (PQ ) of the two lines. Let V, 1 = Voltage and current associated with incident wave, Vi, h = Voltage and current associated with reflected wave, and Va, Iz = Voltage and current associated with transmitted wave. 640 ELECTROMAGNETIC FILEDS AND WAVES: sriwncroe) ph rile va) (TRANSMIT TED) (REFLECTEDIG, Wl + zm} ta in —— He YA wee Fig. 8.18. Reflections at irregularity in a TL. ‘Then from Fig. 8.18 h=l+h (8.89 (a)] and Ww-VtV [8.89 (b)] Vv where i=, B Reflected current is chosen as negative as reflected voltage if considered positive, will circulate a current in the opposite direction w.r.t. J. Hence by putting these values in Eqn. 8.89 (a) vv VM _V-M Zo hb hm or Vo Zp =(V-Vi) Ze or (V4Vi1) =(V-Vi) Ze fromEgn. 8.57 (b) V(Zr-Z or Vi (Zn) =V (2g %) or = USE or Vi = KV (8.90) Thus voltage reflection coefficient K is K Zo (8.91) Further, the current reflection coefficient can be obtained from Eqn. 8.89 (b) V2 VtVyy or by Zp = 1% - Ly = &(1~h) or (A+!) Ze =H U-h) from Eqn. 8.89 (a) = hh (Zp + 2) = I (Zo ~ Ze) _ | {%22e)__(%-% . u-1( B24) =-(S54)s x T= KT (8.92) _ Za Zo, where K= Ze +2 8.27.2. Expression for input Impedance in terms of Reflection Coefficient. The input imp- edance of a T/L is given by Eqn. 8.23 reproduced below r . __,, [ Ze cosh Pl + Z% sinh Pl 2s = 20) 7 cosh Pl + Zp sinh | 8.23) Changing cosh PI and sinh PI into exponential forms, we get T 4, e7Phy bak re a {ete | 5 x TRANSMISSION LINES AND MATCHING NETWORKS 641 [ex “Ty (a+ By + Grr [= (Z + Ze) - A S (Ze + Zo), wehave %=% Dividing numerator and denominator by =g (itKke (8.93) This Eqn. shows that if Z, P! and K are known, the input impedance of the line can be calculated. It is this eqn. which has been utilized in measurement of antenna impedance. STANDING WAVE RATIO ‘When reflections occur in an incorrectly terminated line, the voltage and current vary in magnitude along its length. When the transmission line is not correctly terminated, the travelling electromagnetic wave from generator at the sending end is reflected completely or partially at the termination. The combination of incident and reflected waves give rise to interference phenomenon and thus standing waves of current and voltage along the line, with definite minima and maxima of current and voltage along the line. If the line loss is small, all the voltage maxima or minima can be assumed to have equal amplitudes and under this condition the standing wave ratio may be defined as the ratio of maximum to minimum current or voltage on a line having standing waves and this ratio is abbreviated (SWR) and is generally denoted by a single letter (S) while dealing with the voltage ratio only, the standing wave ratio is also abbreviated as VSWR. Thus Vinax Vann, | _ | toms Venn Vexin The SWR is a measure of mismatch between the load and the transmission line and is the first and foremost quantity calculated for a particular load. Itg_value is always greater than unity when the termination is not correct. But when the termination is correct, its value is equal to unity, if the termination “Bperfectly matched. This isthe-highty desirable siate of affairs.~ If the voltage magnitudes are measured along the length of line terminated in a load other than Ro , the plotted values will appear is illustrated in Fig. 8.19. Ifthe current magnitude is plotted the figure will be similar except that there would be a/4 shift is position of maxima and minima. In Fig. 8.19 (a), maximum and minimum values are labelled while VSWR = or Ss = 8.94 (a, b)] Tesi VOLTAGE——> DISTANCE > (a) Fig. 8.19. (a) SWR on a dissipatiomuens ti (b) line (loss £268) terminated in a load not equal to Ro 0) Standing waves on a line having open or shor circuit termurman 642 ELECTROMAGNETIC FIELDS AND WAVES nodes and antinodes are indicated in Fig. 8.19 (b). Nodes are points of zero voltage or current in the standing wave system and antinodes or loops are points of maximum voltage or current. When the transmission line is a terminated in Ro , it has no standing waves and hence no nodes or antinodes are formed and the line is known as a smooth line. It may be noted that higher the value of SWR, the greater the mismatch between line and load. In practical lines, power loss increases with SWR and so aiso the noise. Hence a low value of SWR is always desirable except when the transmission line is being used as a pure reactance or as a tuned circuit, 8.28.1. Relation between SWR and Reflection Coefficient. As clear from Fig. 8.19 that the points of voltage maxima occur at points at which the incident and reflected waves are in phase and add up directly and the points at which the incident and reflected waves are not in phase, minima occur. Thus, taking only the r.m.s. value rather than instantaneous and measuring from generator end. Vax = |Vi | + 1 Ve | 18.95 (a)] and Vmin =| Vil - | Ve | [8.95 (0)] where V; = rms value of incident voltage and V- = rms value of reflected voltage | Vowx | _ [Vi | + | Vel Now by definition eqn. 8.94, VSWR = | Vel = ivi-twl (8.64) i+ |% or VSWR = V, |dividing by V;, numerator and denominator aid VSWR = feix] t ix from eqn. 8.84 (a) _...[8.95 (6)] Applying componendo and dividendo, VSWR _14/K 1 ~1-1K VSWR -1_1+/K/-1+|K|_ 21K] or VSWR+1° 1+|K[+1-|K]~ 2 VSWR - 1 or |x| = VR= 1 < VSWR < 896(a)] or simply jx} = 44 8.96 (b)] S#1 =lbzen - ae Since 3 (8.91) Thus, reflection coefficient K can be computed, if the load impedance Zp and characteristic impedance Zp of the line are known. Therefore the value of either K or VSWR (or 5’) can be calculated if the voltage maxima and voltage minima are known. K is also calculated graphically using smith chart. It is evident from eqn. 8.96 that at position where the incident and reflected wave terms add in voltage relation, they subtract in the current relation. and vice versa. Therefore, the voltage maximum position is then a minimm current position. The value of minimum current is given by tas = Wd Vel ZL At this position impedance is purely resistive and has the maximum value it will have at any point TRANSMISSION LINES AND MATCHING NETWORKS 643 along the line i.e. Vr Zu = Yess = Wil + Yel gf Le Lvel] ead min | Vil = | Ve Iwit=Tvel Jo | [ve D ~ Al Znax = Z| L+1KE [; ~ 1K] | % | 2mm = 20 x VSWR | (8.97) At the position of the voltage minimum, current is maximum and impedance is a minimum and real i.e. neg = M+ LW x ve 1_|% then Zp = Yon = LVL = L¥el Vi Trax Mil + [Veh Ve Z 1+ Vi =X | tolkKl). 4 ah afte “Tek 8.98) 1-|k| fas. 8.97 and 8.98 will be the values of maximum and minimum impedances. 8.28.2. Location of voltage maxima and minima. If the voltage at any point distance x from the sending end is Vz , then it is equal to the sum of incident voltage ( Vi) and reflected voltage ( Vr). | Vg passes through successive maxima or minima as at certain points which may be designated Ynax (say), the two components add up in phase and give ( Vmax )- Similarly at other points say ( Yain ) when they are in phase opposition, it gives (Vin ) Now fora lossless transmission line & = 0 o P=j— ‘Then from eqn. 8.85 (a), veAe'™+ Be” ie. V, = Ae? + BelBY (8.99) where y is the distance measured from load end Zp. If K is the reflection. Coefficient, expressed in magnitude and direction both i. in polar form, then K=KZ0=|Kle* (8.100) v,_ BelPY Bei? wx Tee eT * 4 gribs wt eeeiving end y = 0 fromegn. 899 K = B/A (8.101) Putting this value in eqn. 8.100, we get 3 =|Kle® ot But = Putting eqn. 8.102 in eqn. 8.99, Vee Ae 4A LK e®. =AeP TL + [K| el®. 2/87) Ve= Ae PLS [K] eFB=-M)| (8,103) 644 ELECTROMAGNETIC FIELDS AND WAVES Taking magnitude only, we get os [Vel = LAT CL + [KR] etb?- 9] (8.104) Since voltages are having maximum amplitudes when the two components are in phase i.e. at values. Qnn (8.105) where ‘The maximum absolute value is given by (from eqn. 8.104) [Vom] = TAT +1 KT (8106) Similarly, the voltage has the minimum magnitude when the two components are in phase opposition i.e. at values y= Yon where 2B Ynin-@=(2n+1)m (8.107) where n= 0, 1.23... Thus the minimum absolute value of the voltage is given by | Vminl = [A] {1 -—[K |] } +=(8.108) Putting these values Of Vnex and Vin in eqn. 8.94, we get Vinx Al(1+[K]] _1+[K VSWR | fe: =Taptr [ei = 11K {8.95 (0)] ‘This is the same eqn. as 8.95 (c) but provides an alternative method for establishing relationship between VSWR and K. However, in this case eqns. 8.106 and 8.108 help in computing the distance of any ‘maxima and minima from the terminating end. The positions of successive maxima or minima are found by putting n = 0, 1, 2...» in egns. 8.106 and 8.108 respectively. The location of a voltage maxima and minisna and the specificationof VSWR completely determines the reflection coefficient (K ) and load impedance (Zp) when the characteristic impedance ( Zp ) and the operating frequency, are known. Eqns. 8.106 and 8.108 are put in alternative ways as slotted line measurements in terms of first maxima or minima from load end. 2m / 0 = 2B ¥mx - 2nm = 2 Yun - 0 n=0 ie. first max. O=2n (=) radians = 360 (2% } degree {8.109 (a)] and @ = 28 Yon -(2n+1) - = Erinn % n= 0 i.e. first minimum 8 Ae - ‘) rad 8.109 (b)] and © = 180° eo - ') [8.109 (c)] 8.29, IMPEDANCE MATCHING DEVICES between an ansmission line in its role as a connecting links between a transmitter and an antenna or antenna and a receiver, affect the efficiency of power transfer. In some cases they may be TRANSMISSION LINES AND MATCHING NETWORKS eS arranged to apportion a total amount of power among a number of loads. Methods of adjusting the line impedance relations toachieve optimum results in these functions are discussed under the head of matching devices. In communication networks, the element of the network should be designed such that maximum power transfer takes place between the source (or transmitter) to the load (or antenna). This is governed by a well known ac. circuit theorem known as ‘Maximum power transfer theorem’ according to which, the maximum power is absorbed by one network from another network, when the impedance of one is the complex conjugate of the other. This means for maximum transfer of power between source and load, the resistance of the load should be equal to that of the source and the reactance of the load should equal to that of source but of opposite sign ie. when source is inductive, the load should be capacitive and vice-versa. When this condition is achieved, it is usually referred to as impedance matching and the methods employed to achieve this, are termed as impedance matching devices. 8.29.1 Principle of Impedance Matching. A radio transmitter is in effect a generator having an internal electro-motive force (e.m.f.) which may be complex as Zq = Ry + iXe {8.110 (a)] According to maximum power transfer theorem, maximum power transfer will take place to load, when it is complex congugate i.e. Zy = Ry — Xe {8.110 (b)] The ultimate load for the transmtitter is the antenna but the immediate load may be the input impedance of the transmission line i.e., Zs = Rs + jXs {8.110 (c)] which is connected between the transmitter and the antenna. Hence for maximum transfer of power from the transmitter, it is necessary that R, = Rs and X, = - Xs {8.111 (a, b)) By Eqns. 8.32 (a) or 8.32 (b), the value of input impedance depends on Zr , Zp and length J. Thus itis possible to choose or adjust the values of these quantities to achieve the desired values of Zs. However, in practice the desired result is not achieved in this way. For example, because the length / of the line may be determined mainly by the necessary spacing between the transmitter and the antenna and hence will not be independent adjustable. The possible range of Zo is also some what practically limited. Lastly, itis usually desired to minimise the SWR on the line, it means Ze must be at least approximately equal to Zo. Hence the impedance generally presented at the input terminals of the line will be t= hh wa(B.112) ‘Therefore, an impedance transformer is then introduced in between the line input and transmitter output to transform thecharacteristicimpedance Zp into the value needed for maximum transmitter power ‘output or optimum performance where best operation from some stand point other than maximum power is required. Further, the actual load impedance presented by the antenna on the transmission line output terminals may not be equal to the desired value Zo. So another transformer may be needed between the load end of the line and the antenna terminals and this one must transform the antenna impedance (Zp say ) into the characteristic impedance Zo. Sometimes these transformers of impedances may consist of coupled coil of wire or of coil and condenser circuits specially at low frequencies. However in the higher frequencies region (Le. in MHz region) these impedance transformers usually consists of sections of transmission lines in the various arrangements. Some of the possible arrangements are ; (i) Quarter wave transformer (Impedance inverter), This will be discussed now. (ii) Stub matching. 646 ELECTROMAGNETIC FIELDS AND Waves 8.29.2, Quarter Wave Impedance Inverting Transformer, Lengths of transmission lines have certain properties which are of special interest for particular values of the length (1 ) or the load impedance Zp. From the Eqn, 8.32, it can be seen that a transmission line is, in general, an impedance transformer i.e, if an Zp is connected at the load end, a different impedance Zs may be present toa source at the input end, From Eqn. 8.32, If ! Then BI = electric length = =z, | Ze cosn/2 + j& sinn/2 O+jZ-1] -- cosn/2=0 % a| Renee + ize sinn/2|~ | 0+j;Ze- 1 sinn/2 = 1 ~o:b_o 25-4,% 3 {8.113 (a)] or B=NG te {8.113 ()) 1 org, Zaz [8.113 (0)] Thus, as clear from Eqn. 8.113 (a), the product of the input impedance (Zs) and load impedance (Zn) equals the square of the characteristics impedance (Zo ) of the line. From this Eqn. one infer shat the quarter wavelength line transforms a load impedance Zg that is smaller than Zp into a value Zs that is larger than Z and vice-versa. Sometimes a quarter wave line is called as impedance inverter for this reason. Ifa Zp is pure resistive, the input inipedance Zs will also be resistive (i.e. real). This quarter wave transformer is useful when it is desired to transform a resistive impedance into a different resistive impedance value either larger or smaller. The desired transformation is achieved by choosing the appropriate value of Zpin accordance with Eqn. 8.113. The desired value of Zp can be obtained by choosing proper value of the ratio of spacing of the line conductors ( S) to their diameters. Because the physical dimensions can not practically have an infinite range of values, the transformation ratio of the 4/4 transformer is subject to the practical limitation. However, itis a very useful device because of its simplicity and the ease with which its behaviour is calculated. Length ! equal to odd numbers of 1/4 will have the same transformation properties but as the length is made larger the sensitivity to a small change of frequency becomes greater. From Eqn. 8.113 (c) input impedance is inversely proportional to the load impedance. Thus if Zp is high, Zs will be low and vice-versa. Also assuming Zpas resistive, if Zr is capacitive, then J Lg nte—maini T/A, oj R Me L Fig. Zs will be inductive and vice-versa. Thus depending upon the Zp, a 4 transformer line acts as step-up or step down impedance transformer and that is why it is impedance inverter. An excellent example, is matching of a dipole antenna with resistive input impedance Rs is matched with a main transmission line of fig. .20, Matching of a/2 dipole characteristic impedance Z} by means of a 1/4 transformer having with 2/4 transformer. characteristic impedance Z = VZ%0* Rs- ‘A M4 transformer, however suffer from the disadvantage that itis sensitive to change in frequency, because at a new frequency (i.e., wavelength) the section will no longer be a 1/4 in length. sZo_ 2G TRANSMISSION LINES AND MATCHING NETWORKS. 647 830. STUB MATCHING the A section of transmission line can be used as matched section by inserting them between load and Source, as seen above. Besides it is also possible to connect sections of open or short circuited line Known as stub or tuning stub in shunt with the main line at a certain point (or points) to effect the matching. The matching with the help of tuning stub or stub is called stub matching and is having the following advantage : (®_ Length (1) and characteristic impedance ( Zo ) remains unchanged. Gi) Mechanically, it is possible to add adjustable susceptance in shunt with the line. A stub matching is of two types, the single stub matching and double stub matching. _ 8.30.1, Single Stub Matching. A type of transmission line frequently employed for single stub matching is shown in Fig. 8.21. The principal element of this transformer is a short circuited section of line whose open end connected to the main line at / a a particular distance from the load end, where the eineur input conductance at that point is equal to the characteristic conductance of the line, and the stub length is adjusted to provide a susceptance which is equal in value but opposite in sign, to the MAIN TRANSMISSON input susceptance of the main line at that point, so a ’ that the total susceptance at the point of attachment is zero. The combination of stub and line will thus ~~ ~~}.<=~~ ==> present a conductance which is equal to the characteristic conductance of the line i.e, main length of H-F. transmission line will be matched. Let the characteristic admittance of the transmission line be Yo, which is terminated in a pure conductance Yg as shown. Since the stub is connected in parallel with the main line so it is better to do the calculation with the admittance rather impedance so that it can directly be added up. When Yr # Yo, as we know standing waves are set up along the line. As we transverse the line from the load towards the generator, the input admittance (looking towards the load ) varying from a maximum conductance through 2 parallel combination of conductance and inductance, a minimum conductance —a parallel combination of conductance and capacitance — and again maximum conductance to minimum conductance and the cycle reparts every 1/2. If the line is transversed from the point of maximum (or minimum) conductance to that of minimum (or maximum) conductance, then obviously there will be a point at which the real part of the ndmittance is equal to the characteristic admittance. If now a suitable susceptance is added (by adding a proper length of short circuited or open circuited Tine known as tub) in shunt a this point so a8 to get etnesonance with the susceptance already existing, then upto that point matching has been achieved. ‘Although there exists a mismatch between this point and the load, yet this mismatch over a short length is of insignificance. Hence it is necessary that the stub should be located as near to the load as possible. Further, the characteristic admittance of the stub so connected in shunt should be same as that of the main line, The mathematical relation for the location of stub ((; ) and the length of short circuited stub (lt ) can be started from the input impedance equation of a transmission line e.g. Zx + Z tanh Pl Baw eto Z + Zp tanh Pl Now converting impedances into admittances by putting Loan Fig. 8.21. Single Stub Matching. 648 CLES TROMAGNETIC FIELDS AND WAVES Yo = + ie, Characteristic Admittance. % 1 A Ye = k ie Load Admittance and. ‘Ys = 7- He. Input Admittance, (ty, + ae tanh Pt) .. mee (Z + Ze tanh Pl) _ yy Yo we Y= 75" a) Ca + wah PT)” O ( ann Ye * Yo Yew Yo (te+ Yo anh PI) way (%o + Ya tanh PI) (Yr + $¥o tanh Bl) For ahigh frequency linea = 0, and P = ‘jBso that ¥s= Yo Cy + j¥e tanh Bl) Using normalized Admittance i.e. a = Y, = Normalized input admittance. a = ¥, = Normalized load admittance. (gesmen) ‘We obtain, ~ (14 He = 6) (iy, + j tan BL) (1+ 7%; tan Bl) he (8.115) on rationalization Eqn. 8.115, we get Y= (%, + jtan Bl) x (1 =i% tan BL) (1 + j¥, tan Bl) “(1 ~ j¥, tan Bl) ~i¥,? tan Bl + j tan Bl - 7 ¥, tan? Br 1+ (¥, tan Bly? y, = =F tant Bl) + j tan BC ~ 9,7) 1 + (¥, tan BL)? * Ys = Gs +7 By = Ye (1+ tan? Bl) + jtanBi(a — y,? U1 + (¥, tan Bly] (8.116) y,-2 Equating real and imaginary parts we get Gs = %, (1 + tan? BI) 1+ (¥, tan Bly? Bs = 2nBl_ (1 - ¥,*) 1+ (¥, wn Bly ¥s= Gs +jBs=1+j0 and ¥s= Gs =1 Bs=0 [8.118 (00) (8.117 @)) and [8.1170] But for no reflection o[8.1170) TRANSMISSION LINES AND MATCHING NETWORKS 649 Thus stub has to be connected where there is no reflection i.e. where real part of Eqn. 8.116 e.g. Eqn. 8.117 (a)] is unity. Thus pater dy Ge = 1 = YC + tan? Bly 1+ (¥, tan Bie? or 1+? tan? Bl = ¥, (1+ tan? BL) or ¥, tan? Bh (Ye —1) = r= ?a,st-t ~ 1 or tan Bu= y= ye or tan Bl, = 7 eof 8.119 (a)] Yo a 4 [% 1 1 = tant af 20 ed art or 8 ‘any Ye oo | pa or ba jw This eqn. gives the location of the stub from the load end as shown in Fig. 8.19. Now the susceptance at the point of attachment of stub is given by Eqn. 8.117 (b) Bs _ tanBls (1 - ¥,7) Yo 1+ (Y¥, an BLY VE(-¥) Bs _ Y ¥ % B= Yo YR Yo 1+ Be Yo ¥ to (x) (%- YR) Yr | (Yo~ Ye) (Yo + Ye) Bs - ho ~ Yo + Yi Yo i+ % fy | 8 Bs _ Yo\ (o-¥e) g a= | (¥% - Ye) (8.120) Yo Ye Y Yr i f stub is this Bs . This tance which should be added at the point of attachment o 5 an be coeleed einer by ‘an open circuited stub or short circuited stub. The desired length of the stub a ) which will provide this susceptance Bs , is readily computed with the help of fundamental Eqn. I Vr = Vs cos Bl - jo Is sinBI Since the short circuited stub has the advantages of (i) Less power radiation and (ii) Effective length variation is possible by a shorting bar, therefore, a short circuited stub is invariably used. For a loss-less short circuited stub Ve = 0. So the short circuited impedance is 650 ELECTROMAGNETIC FIELDS AND WAVES, or short circuited it is = 1 = —t— xf circuited admittance is Y% Z ~ 7% tanBh x j or ¥, = Gi + jB, = — j Yo cot Bl (8.121) Susceptance at the point of attachment of short circuited stub is obtained by equating real and imaginary parts from eqn. 8.121. Thus G and Br = ~ Yo cot Bl (8.122) where 1, = length of the short circuited stub Now at the point of attachment, the sum of the line susceptance and the stub susceptance must be zero i.e. B+ B,=0 or V (Fp | (2 m+ [- Meco Bu] <0 or pi =( 25%) z = (o= Ya) Te or tan Bi = SPOT he A tan ym EF = x tan? jae (8.124) This is the equation of length of a short circuited stub required. Stub matching is almost invariably used aia fairly higher frequencies i.e. in S.W. range of broadcast band. The basic idea in using stub is to avoid reflections reaching the sending end i.e. source. 8.30.2. Demerits of Single Stub Matching. The single stub matching suffers mainly from the two main disadvantages. ; (a) The range of terminating impedances which can be transferred is limited. If the terminating impedance (i.e. input impedance of antenna, say) changes, then it becomes essential to adjust the position and length of stub as well. It is easier for open wire lines but is inconvenient in case of coaxial lines. (b) Itis useful for a fixed frequency only because as the frequency varies, the position of the stub has. to be varied. The change of susceptance, however, does not present any problem because the shorting plug can be moved to the required position. Hence, the single stub matching system is a narrow band system. 8.30.3. Length (Z, ) and position (L;) of stub in terms of Reflection coefficient K. It is often convenient to determine L, and L, when Kis given. To arrive at a relation, let us start with the eqn. 8.93 . (+ Ke?) L=% (1K e2”) (8.61) For high frequency transmission lines, the attenuation o = O and hence P = j B. Further, the reflection coefficient X is a general complex quantity and can be written as K=(|K|e/* where - “ @ = Angle of reflection coefficient K TRANSMISSION LINES AND MATCHING NETWORKS 651 so that ‘Therefore eqn. 8.93 can be written _y, Lt ike? x eho Tee-tB, Zs = Zy e* x ¢ 1+|Kle U1 = [kK] oF? x HOF = Ey |p eA] | “OH As stub is connected in parallel, so itis better to use admittance 1 Ys = 3, = Input Admittance = Gs + j Bs Gs = Input conductance and Bs = Input susceptance 1 Yo= 7, = Go + jBo ‘Thus eqn. 8.125 can be written as Yow den DLL = LK | olto-280 1a [Kp ete 2 gedbet (8-126) = = & Zs ZH (r+ K|e-2P)) ti + | K | el It is assumed here that characteristic impedance is resistive Le. = 38D] Z= Ro= q_ where Go = Characteristic conductance ‘Now eqn. 8.126 can be converted to polar coordinate as ef = cosO + HieG Ys = Ge cos -2 + jsin(@ - 2 =“ 0 Tete eos(@ = BBD + Jain 260) _ ¢, LU = 1K] cos (9 = 2 jsin(@ - 2 Ys = Gy aN aaa el gaine = 2800] Let o = 0-21 [1 = [K] cos 9’ = LK Lj sing’ uae ve= 00 TS Rae Further, let | K|cosg’=A and | K| sing’ =B 1A) -jB) ‘Then ¥s= Goi a) +78) Now ranaizig wehave , 1-A jB 1+A)-jB Y= Go tp ay + 1B) © (C+ A) JI 1A) (1 +A) = {BC +A) -jB UL - ADH PB ty 5'Go T+ ay - Fe) pi2-arjB{L+At1-A +P] _ 4 (1-A-2j8-H] = Go [12+A7+2A+B"] [+A +B 424) Putting the values, we get Yo = Gy LL= LAE os’ of = 21K] j sing’ - [KP sin? 9°) [1+ | KP cos? of + [KP sin? @” +2 [| cos’) =-G U= KE [cos? o + sin?o’| - 2] K | jsing’ (1 + [KP [cos? g” + sin? g’] + 2 |K] cos 9") 652 ELECTROMAGNETIC FIELDS AND WAVES ¥s = Gp U=1K P= 2 1K) sing” (8.127) [i+] KP +2 |K | cosg’] Got [1 - [KP - 2) K|sing’]] Ys = Gs + jBs = ‘ POs TT + [KP + 2[K cos 9] Equating real and imaginary parts z @=-—AUl=1e) _ [8.128 (a)] 1+ 2[KP +|K| cos” and few a8 Le] see). nf8.128 0)] [1+|KP +2 |K[ coso’] At the point of attachment of the stub, for no reflection to take place Zs = Zo at | = Js LLL. Gs_ or Gi iw G7! mink Hence from eqn. 8.128 (a), we get. 1 ——_ 11 1+(|KP +2|K| cos(@ - 2B ls) or 2|K| cos ( - 2B) =-2|KP or cos (9 - 2Bls) = - [KI or ( - 2Bi) = cos! (-|K]) (8.129) But cos! (—|K|) = cos? | K| - © Hence (@ - 2Bi,) = cos” |K| - 7 or 2Bls=o+n-cos'|K] 1 a y= Sta oe 1k 1) or 1 = Fe (@ +m cos! |K]) per unit (8.130) Again, from eqn. 8.128 (b) the shunt susceptance on a per unit basis is given by Bs _ [-2|K| sing’] _ -2| K | sin (9 ~ 261.) Go 1+|KP+2|Klcosg’] 1+|KP +2] K | cos(@ — 2B) The input susceptance of the line at the stub location nearest to the load is obtained by putting eqn. 8.129 into eqn. 8.130. Thus Bs _-2|K|sin (cos |-K|) _-2|K]|sin(cos"'|K|-m) _-2|K|sin(cos”"|K| ) = = 8.131 Go T+ |KP+2[K|{-K]} 1+|KP+2]K|x|-K] 1+] KP-2| KP een In order to find the value of sin | cos (| K |) |. Letus put cos! (| K|)=8 9 or [K | = coso io sind = V1 — cos? @ = V1 [KP Hence Bs ___-2|K|sin@_ _ -2|K| V1 -| KP Go 1+ |KP-2|K I-1KP TRANSMISSION LINES AND MATCHING NETWORKS 653 ox Bs = Go -2/K| V1 -{[K' = -—2|K eo Go AED (8.132) Now, if B; is the susceptance of the stub and B, the line susceptance, then in order to cancel the susceptance, both must be zero at the point of attachment, Thus tate But the susceptance of the short-circuited stub is given by eqn. 8.122 i.e. By = Go cotBl, where 4, = length of short-circuited stub or Go cot Bi = Gy { -PLEL} 1 __ 24k _ V1= [KE tan BE VE pap B= oT Ry or By = B= or Bi = ton MERTRE gy eb ae! vi= [KE 2|K| B 2|K| or (8.133 (@)] From eqn, 8.128 (a), it can be seen that & Gs__ KP ¢ ode Go” 1+ |KP + 2|K| cos(@ - 2B) Al t This value of & is maximum when MAN TRaNsMSsON —_/ 2, ]uox0 denominator is minimum ie. when cos (9 - / 2Bl,’) = — 1. Thus cosine terms should be -1 for ay 7 maximum value of ’ . Hence // cos (@ - 2BL’) =-1 ( Ja @ - 28h’ = Sincureo \/ or -2BL = STUB y or 2B Fig. 8.22. Location of Stub. or uw (o+m) eo[8.133 (6)] Ata distance ly’ from the load the value of Gs _ 1-| Kf ~ C+] KIVG-|K])_1+1 KI G@ 1+(KP +21 K1(-1) G-1KIy 1-|KI Gs _1+1KI _ yowr = G ~1-IKI 654 ELECTROMAGNETIC FIELDS AND Waves 1 Gs ow Gseeet AB. or Eas 0 Gs= 7" Be 8.134) oe 1 dGebet Bxa=s and Go = 7 = Re . % or 4 =3 (8.135) This is the point of maximum & which is recognised as a point of minimum voltage ata distance 70 of |, from the load. At a distance /, from the load, the value of 2 = Lor Gs = Go. Thisis the point where stub is attached. The value of /, has been calculated at eqn. 8.130. Obviously, the value of distance dis given by _O+n = cos"! ([K 2B 28 -=-[8.136 (a)] VSWR = 1 or a } {8.136 (6)] This shows that stub should be connected at a distance d measured in either direction from the voltage minimum point A. Normally stub is attached towards load side of that minimum which is nearest to the load voltage standing wave ratio which is.existing before the attachment of stub, we may express conveniently the length of the stub in terms of VSWRas follows. From eqn. 8.133, han? VE=LK 25 OTK VSWR -1_ S-1 VSWR+1° S+1 be But [KI= Putting the values, we get A 4 Ge - h=o, 9 Sat "25 S#1 NS 4¥ie2s-8-1425 2(S-1) metres This is the length of the short circuited stub which can be placed d metres towards the load from a point at which a voltage minimum existed before attachment of the stub. The susceptance of the line at d is cancelled and the line appears to be terminated in a pure resistance of value Ro at Pp lecs This way there will be a smooth line between the generator and the point of attachment stub, ‘TRANSMI: ISSION LINES AND MATCHING NETWORKS a3 Alternati The i Sub MAY also be attached d metres towards the source from the voltage minimum (A A). The sign of 7 load. In this case, stub a {8 reversed on this side w.r.t. the sign for the location nearer the = ie = (8.138) Normally a short circuited stub is preferred in'compari ircuit ison to an open circuited stub because of greater ease of construction and Secondly because of lower bas of energy due to radiation. 831. DOUBLING STUB MATCHING __ The two disadvantages of the single stub matching are overcome by using double stub bt. matching as shown in Fig 8.23 in which two short. ~ $ a circuited stubs attwo fixed point usually A/4apart source Pie }ueso are utilized. Although positions are fixed but their lengths are independently adjustable. The double ~~" ~ stub matching provides wide range of impedance matching. In a single-stub impedance matching stub is located at a definite point on the line. ih a This requirement frequently calls for placement Fig. 823. Double stub matching. of the stub at an undesirable place from mechanical point of view. For example, in case of coaxial line it is not possible to locate the location of a voltage minimum without a slotted line section. Thus the placement of single stub at an exact requirement, creates difficulties. In the case of single stub matching two adjustable parameters viz. length and position of the stub. Alternative way to have the required adjustments is to use the two stubs for matching as shown in Fig. 8.23 in which positions of the two stubs AB and CD are arbitrary but the two stub lengths are adjustable. The spacing between two stubs is 0.375 A. or V4 . Half wave length (4/2) spacing is avoided as it places the two stubs in parallel, as result only one effective adjustment is available. The same difficulties are encountered if the two stubs are close together. The separation of 0.375 A or 3/8 A is an optimum. The double: stub matcher as shown in Fig. 8.23 is usually employed for coaxial microwave lines. In double stub matcher, two variable are available which provides a very good matching. If the first stub whose length is 4, input admittance is then given by eqn. 8.115. is located at AB at distance of I,, from the load, the normalized Y, + j tan Bs, (¥s)ao = 27 ¥, tan Bla, (8.115) Rationalizing, we get Y; + j tan Bly © 1 —j¥r tan Bly, _ ¥,+jtanBls,—j Yr tan? Bi, + ¥, tan” Bly, (Ys)ae = TSF, tan Bh, 1-7 ¥; tam Bl 1- CY, wan Bi, Ye (1+ tan? Bly) + jtam Bly (1 = Yr) _ Yr sec* Bl, (1-¥#) tam Bl = 1+ ¥ tam Bb, 1+ tant Bi, 1+ tan Bl, (¥s)aa = (Gs)as + j (Bs an -=(8.139) 656 ELECTROMAGNETIC FIELDS AND WAVES: ¥, sec? Bi, (1 — ¥#) tan? Bl, where (Gs )as = a = . _ dap TP a and (Bs)an Ta wan Bh, {8.140 (a, by] and se Yo = Normalised input admittance when a stub having a susceptance By is added to this point AB, then new admittance will be say (¥s)co = (Gs )an = co + j (Bs co (8.141) Because only the susceptance value is changed by the addition of the stub, while the conductance Part remains unchanged i.e. ( Gs Jas = (Gs )co . Then ( Ys )cp should be such that admittance ¥ at CD is equal to 1 + j Bz. The stub length at CD is adjusted such that the new value of ( ¥s )cp is unity to effect the matching. For smooth operation of the line, the input admittance of the line looking towards the load at CD should be ( ¥, = 1) or Ys z = lor Y= Go (8.142) or the line should appear terminated in its characteristic impedance at that point. Hence the point CD should be at a location on the line having a per unit admittance of ¥s = 8. (8.14: Ya Ga ltjh (8.143) It may be noted that two stubs are usually at fixed points normally separated by a distance 0f 0.375 2. The stub AB nearest to the load is adjusted to make the real part of the admittance at the points CD equal to the characteristic conductance of the line, in absence of the second stub (AB). This stub (AB) is then adjusted to produce zero susceptance at the points CD. Double stub matcher is usually connected between the load and the main transmission line to ensure the shortest possible length of mismatched line. The mismatch occurs between CD and the load end. In order that this mismatch portion on the line is minimum, the total distance /,, + [,, should be as small as possible. This is the-reason the stub AB is kept nearest to the load. The two stubs whose lengths are individually controllable are shunted across the line near the load as shown in Fig. 8.24. This arrangement has the advantages that trial and error adjustment of the impedance matching system can be made without the necessity of providing a connectioa that can slide along the main transmission line which makes it suitable for coaxial transmission line. However, there is a disadvantage of the system that the range of the load impedances that can be matched to the transmission line is limited. 375% ADJUSTABLE SHORT PLUNGER To TO LOAD GENERATO tay ts, Fig. 8.24. Double Stub impedance matching system. TRANSMISSION LINES AND MATCHING NETWORKS 857 By double stub matchi ing, it is possit ; under all conceivable conditione ot viene © reduce VSWR below 1.2, If almost perfect matching ns of VSWR and load impedance Zp is required, a triple stubs, adjustable Is, and Is, to calculate doubl le stub determined withthe help of smith a Problems analytically. However, the value of fs, and Is, may be ‘mith chart. The length of stubs are in the range of, 4, = 0.348 % and ij, = O.11 A ofR.144 (b)] 8.32. TYPES OF HIGH-FR The are many types important types are : (Coaxial cable ; Gili), Twin lead or ribbon lead. 8.31.1. Coaxial cable. If one wire of open wire line is placed inside a larger hollow concentric conductor the arrangement is called as coaxial cable. Itis called so as both the conductors (outer and inner) have the same axis (Fig. 8.25). In practice, the central conductor is held in position accurately by insulating material which may be a solid core, disc or breads strung along the axis of the conductor. Equal and opposite current flows in inner and outer conductors of a coaxial cable. The presence of outer conductors provides a shielding effect and prevents interference from external source and radiation losses at high frequency. Coaxial cable is of many different forms and is used upto a frequency of 3 GHz. Flexible form of a coaxial cable Fig. 8.25 (b) is generally used for connecting a transmitter, receiver to the antenna. It usually have a continuous dielectric, the inner conductor is stranded and the outer conductor is in the form of braided wire to provide necessary flexibility. EQUENCY TRANSMISSION LINES (FEEDERS) of ELF. transmission lines which are in general use, of these, the most (ii) Open wire ; OUTER CONDUCTOR = gure” - OUTER conouer INNER crop COVERING (ORAIGED SMES INNER ‘CONDUCTOR POLYTHENE INSULATOR (a) (b) Fig, 8.25. Coaxial Cables. The characteristic impedance Z of a coaxial cable in terms of conductor size and spacing may be obtained, if the inductance and capacitance per unit length of the cable is known. As already deduced, the inductance and capacitance of a cable is given by tog. 2 Hm and vim | Laon 8 ad \ where D = Dia. of outer conductor ; — d = Dia. of inner conductor. i = Permeability of the dielectric between inner and outer conductors € = Permittivity of the dielectric. Further, at high frequency _— B= Ho Br Fe loge D/d m= = YA = VE - tog? Did ands = 64 Fr A ic Ine ane loge D/d 858 ELECTROMAGNETIC FIELDS AND WAVES 2 = \J— He hog, (Dia ice = 1 Tere Nae ES toe (D/4) | Porsiye = 18 an x 107 jo = 40 x 107 H/m 4 an _ p/m = —}_ = V4 tog, (a/dy| €0= 8.854 x 107 F/m see 36m x 10° = 3600 x 2.3 logio D/d = 60 x 2.3 logio D/d Z = 138 logio D/d ohms {8.145 (@)] {f the insulating material between annular space is of relative permittivity €,, then formula is of the form = 2 = logio D/d ohms s-L8.145 (b)] It may be noted, however, that a coaxial cable is an unbalanced line, with respect to earth, 8.32.2. Open wire lines (Fig. 8.26). It usually consists of two copper or copper alloy conductors, spaced a few cra apart and fixed by spacers i.e. insulators. A two wire open lines are easy and cheap to construct. Since the insulation is almost air, therefore, dielectric loss is extremely small. An open-wire line is a balanced line w.r:t. earth and this is the advantage of this system. How- + ever, use of open-wire lines becomes undesirable SW spacer above the frequencies of the order of 100 MHz arene. because of the radiation losses which increase, if Fig. 826: An open wire lines frequency increases. Sometimes four wire lines are also used which are placed at the corners of a square and has the advantage of lower characteristic impedance in comparison to two-wire lines which reduces the insulation losses. conodcTors The characteristic impedance of an open wire line at high frequency is given by where £, C are the inductance and capacitance per unit length. The characteristic impedance of an open wire line in terms of spacing and size of the conductor is obtained putting the values of Land C. As shown Previously tha: capacitance of a pair of parallel wires of spacing s and radius r (where >> r) is given by mE | = F/m | leee sr where € = € & = Permittivity of medium in which the wires are placed. £, = Relative permittivity and &o = Permittivity of free space = 8.854 x 10°? F/m 1 * 360 x 10° Also the inductance of a pair of parallel wires is given approximately by F/m ISM TRANSMISSION LINES AND MATCHING NETWORKS = where H = Ho Hr = Absolute Permeability Permeability of space e 0 =4 mx 107 H/m ; yy = Relative permeability F loge slr 20 = Fang atime = AV (one? ey bar = 1 for air loge s/r nme, lan x 1o7 =a a (loge s/r) = e loge (s/r) z= & 36m x 10” 120 2 = FE loge (4/r) = 8 logo (s/r) or Z = 276 logo Although a symmetrical two wire line is @ balanced pair and does not radiate any appreciable Power at low frequencies, but radiation losses become excessive at frequencies where the inner conductor Spacing is comparable to the wavelength. Besides, with increase of frequency, it becomes increasingly difficult to maintain the high degrees of balance required to shield the parallel wire line from external interference and it becomes necessary to provide an external shield. Of course these two difficulties are overcome by the use of a coaxial cable. 8.33. BALANCE TO UNBALANCE TRANSFORMATION Sometimes grounded antennas or monopole antennas or Marconi antennas are fed through a two-wire transmission line and an ungrounded antenna system that is symmetrical w.r.t. ground is fed through a coaxial transmission line. An open wire transmission line, being symmetrical w.r.t. ground is a balanced transmission line while coaxial cable is an unbalanced transmission line. In either ‘case, whether Power is to be delivered to a grounded antenna through a two-wire transmission line or conversely coaxial cable is used to deliver power to an ungrounded antenna, it becomes necessary to convert between a balanced system which is symmetrical w.r.t. ground and an unbalanced system in which one side is grounded (i.e. not symmetrical w.r.t. ground). It is because a two-wire transmission line (a balanced transmission line) feeds grounded antenna (an unbalanced antenna). Likewise an unbalanced coaxial cable feeds an ungrounded antenna which is balanced antenna. cs ++ (8.146 (a)} ife,=1 [8.146 (b)] A method used to effect such a trans- eurcrrostane formation is to employ a transformer shown in SHIELD Fig, 827. Here the winding associated with the wearer balanced system is symmetrically arranged w.r.t. ‘Cyt @ grounded electrostatic shield so that stray capacitances enevitably present will not introduce unbalance. ig. 8.27. Balanced to unbalanced transformation with the help of a tuned transformer. ELECTROMAGNETIC FIELDS AND WAVES 660 8.34. BALUNS . | ‘Stands for Balancing unit. At frequencies high enough for resonant lines to be practical, arrangements of the types shown in Fig. 8.28 are employed. They are known as balun. A balun is an impedance transformer designed to couple a balanced transmission line and unbalanced transmission circuit (or antenna). The impedance transformation is accomplished generally with convertional techniques. However, the conversion between a balanced system and an unbalanced system requires special techniques. The operation of the balun can be understood by considering that the voltage is applied to the balun from balanced side. Since this voltage is applied across the conductors A — B of the coaxial cable, therefore, it is transferred to the coaxial system without change. At the sametime, the fact that conductor B is an extension of the outer conductos D of the coaxial cable does not introduce an unbalance in the balanced system at conductor B because when the sleeve is exactly at /4 long the impedance across B—C approaches infinity. Thus sleeve C acts as an extension of conductor D and remains at ground potential, while point B is free to assume any potential the balance system desires it to have. Of various other basic types of baluns possible another type is shown Fig. 8.29. ¢__SUEVE \ Ly we cicaaasl Fig. 8.29. Half-wave-line balun. Fig. 8.28. Illustration of arrangements for transforming a | balanced system (i.e. symmetrical w.rt. ground) 10 an unbalanced system (in which one side is grounded) ie. balun. 8.35. BALANCE AND UNBALANCE LINES A balance line consists of two parallel conductors which are separated a fixed distance apart by some low loss material. When the R.F. Currents in each wire of the line are of equal magnitude and opposite phase the field around the line is small anda very minimum of power escapes from the line in the form of radiation. On the other hand, the electrical field surrounding a line having unequal or improperly phased currents will be large. The field of the line may interact with that of the antenna, which will distort the pattern of the antenna. In any event, the radiated field of the line may be considered to be valuable power loss before it reaches the antenna. Since there is a minimum of solid dielectric between the wires, the dielectric losses of a good two-wire balanced transmission line are low in comparison to a coaxial line having greater amount of dielectric material. An unbalanced line is a Coaxial Cable which consists of a wire inside a tabular outer conductor. ‘The inner wireis insulated and supported by a solid and continuous dielectric material. The outer conductor is usually made of a flexible, metallic braid. The outer conductor keeps the radiation from inner conductor minimum. All the fields exist between the annular space between inner and outer conductors. This makes coaxial cable a perfectly shielded line having no external radiation. Like balanced line, the currents in the inner and outer conductor and the shield are of equal magnitude and opposite phase. The shield current flows only on the inner surface of the shield. The outer surface of the shield is ‘cold’. The higher value of dielectric loss of the coaxial unbalanced line is compensated by the fact that coaxial cable has no radiation loss, as does the balanced line. TRANSMISSION LINES AND MATCHING NETWORKS 661 Example 8.1. Calculate the characteristic impedance, propagation constant and velocity of propagation at 400 ke/s for a transmission line having L = 0.5 mH /km, C = 0.08 i F and negligible Rand .G. (AMIETE, A & WP May 1969) Solution. Z, = L=05 x 107 H/km C = 0.08 x 10% F/km = = 2236 _ 79.08 ohm Ans. * 2.828 - 2" 7 P = jo VLC = jB = j 2 x 3.14 x 400 x 10° VO5 x 10 x 0.08 x 107 = j 6.28 x 4 x 10° x YO.0d0 x 10 = j 25.12 x 10° x VO40 x 10 = j 25.12 VO40 = 725.12 x 0.6325 per km = j 15.8884 per km = j 159 per km | P | = 159 rad/km Ans, and ay, = Dadar 1 = 1 = 1% 108 2B Vic Vosxio*xo08x10* Voaoxi0™ 040 0.6325 vp = 1.583 x 10° km/s = 1,58,300 km/s Ans. Example 8.2. A transmission line has series inductance of 0.56 millihenry and capacitance of 0.1 microfarad per km.If the losses due to conductor resistance and insulation leakage are negligible, calculate the (i) Characteristic impedance, (ii) The phase velocity. (AMIETE A & WP, June 1974) ‘ aph i Solution. Zo = WG and wy = Ee a a 3 me [956x107 g _ ase Riot » b= 056 x 10° H 0.1 x 10 C=01 x 10¢F 0.56 x 100 x 10 _ 1 2 _ 7.483 2 =) EO 10" = 79 V55 x 1 = > x 10 from logarithm table V56 = 7.483 = 0.7483 x 10° = 74.83 = 75 Q Ans, 1 L and % = Tie > Jose xo? x01 x 10% SYS tots 10° 1 from table 55 = 0.1338 1 == SS = ~ Tose 10" 6 7.483 100 10 = 0.1338 x 10° = 1.338 x 10° km/sec Ans. Example 8.3. A high frequency transmission line consists of a pair of open wires having @ distributed capacitance of 0.01 \ F per km and a distributed inductance of 3 mH per km. What is the characteristic impedance and propagation constant at f = 10 MHz? (AMIETE, A & WP, June 1981) Solution. Given L = 3 mH/km = 3 x 10? H/km C = 0.01 .F/km = 0.01 x 10° F/km f = 10MHz = 10 x 10° Hz 662 ELECTROMAGNETIC FIELDS AND WAVES . a=VE = a = V300%10" = 10° 30 = 100 x 5.477 = 547.7 ohms Ans, 01 x =o ViC =2 0x 10 x 10° VOL x 10x 3 x 107 =2 nx 10 x Yoon x 10° x 2 10 Lan x 107 x 10° x =anx 10 30 an x 10 30 = 20m x 5.477 = 20 x 17.20778 = 344.15560 = 344.156 rad/km Ans. |P Example 8.4, A low loss transmission line of 100 ohm characteristic impedance is connected to aload of 200 ohm. Calculate the voltage reflection coefficient and the standing wave ratio. CAMIETE, A WP June 198) Solution. Given Zp = 100 ohm; Ze = 200 ohm; K = ?; VSWR = Ze-Z _ 200-100 _ 100_1 = 300 + 100 ~ 300 ~ 3 4" By formula, K= and VSWR = = 2 Ans. 4. 3 i 2 3.3 Example 8.5. A transmission line has the following constants, R = 10.4Q, L = 3.666 mH, C = 0.00835 1 F and G = 0.08 1 mhos. Calculate Z, ct, B, and vp at @ = 5000 radians sec. (AMIETE, T/L Nov. 1963) Solution. Series Impedance Z= R +j @L = 10.4 +j 5000 x 3.666 x 10° = 10.4 +j 18.33 : =V¥(104)+ (18.33 > Z tan! 4833. = = 2105 2 604° Similarly, shunt admittance ¥ = G+ jw C=0.08 x 10° + j 5000 x 0.00835 x 10 = 10° [ 0.08 + 41.75 } ¥ = 10% VOB) + (ALIS Z tant 41:75 7 B = 4177 x 10% / 89.9° merit Zo 21.05 260.4 mV EN gion 7 41.77 x 10° 2 89.9 Also P=VZ-Y= = 224.5 / - 14.75" 21.05 2 60.4° x 41.77 x 10% 2 89.9" [ 1.3232 + 06 2 log P = 10° Z 75AS* = 107 [1.4719] 275.15" P = 10° A’ log 1.4719 275.15" = 0.2961 x 10% x 109 2.75.15" P = 0.02961 275.15 ° = 0.02961 [ cos 75,15° + jsin 75.15" ] = 0.02961 [ 0.2554 + j 0.9681] = 0.007561 + j 0.02863 = @ + /B (say) Thus = 0.007561 neper per km — and B = 0.02863 rad per km. or = 2 2 SU? = 1.746 x 10° km/sec”=-1,74,600 km/s Ans. 21.05 x 41.77 x 10% OEE TRANSMISSION LINES AND MATCHING NETWORKS 663 Example 8.6. An open wire transmission line having a characteristic impedance of 600 Q is terminated by a resistive load of 900 Q. Calculate the standing wave ratio and give an arrangement for single stub matching. (AMIETE, A & WP May 1970, Dec. 1972) Solution. Given Z = 600Q; Zp = 900 Q; VSWR=? Now 1 1«> vswr = L+1Kl_ 5 1=(kI" 7a 5 Arrangement for single stub matching is shown in Fig. 8.19. = arf A tg \[ 2B b= on tan and uaa, tan Ze -Z ok tan! = oe et Fog tan! VES = yo tant 1.2047 2 = % 50.74" X55 = 7gg X 2537" = 0.1402 Ans. 900A japet | 540000 i,t 734.846 an 300 ~2n"" 300 of tenet ok +x 195. tan 2.4494 => x 67.75" x 180 ~ 300 A =0,1881 2 Ans. Example 8.7. A loss-less transmission line having characteristic impedance of 600 ohms is terminated by a resistance of 300 ohm. Calculate the voltage standing wave ratio in the line. (AMIETE, A & WP May 1972) Solution. Given Z = 600 Q ; Zx = 300 Q, VSWR =? = = % _ 300-600 _ -30_ 1 i Now K-74 2 30+ 60 9007-3 % IKI=3 141 4 eitlRL. 23. 3d TE pon 27372 VSWR=2 Ans 3 3 “Example 8.8. A transmission line of characteristic impedance 600 ohm is terminated by a reactance of j 150 ohm, find the input of a section 25 cm long at a frequency of 300 MHz (Smith chart may be used). [AMIETE, (T/L) Nov. 1973] Solution. Given Z = 600 ohm; Ze = j 150 ohm - 25 1 1 = 25 cm = 755 m= 4 mf = 300 MHz 664 ELECTROMAGNETIC FIELDS AND WAVES Hence input impedance Z, is given by cosh 2/2 + Zo sinh n/2 1 600 x 600 e-} zen | Zeca sas rae laa] Bo og sy ny 2m 1 ohm An Example 8.9. A dissipation-less transmission line whose characteristic impedance is 200 ohm is connected to a load of (100 +j 100) ohm. The frequency is 300 MHz. What is the input impedance of the line, if the length of the line is 25 cm. (AMIETE, A & WP June 1979) Solution. Given Z, = 200 ohm; Ze = 100 + j 100; f = 300 MHz Lm; t=25em=4fmiZ=? a Ae , _ B _ 200 x 200 Vide example 88, 2 = 7° = 799 + 7 100 200 x 200 200_ x 200 200 x 200. _ “ \Z1= Too0j+ caooye ~ V2 x (100)" 100 V2 = 200 x V2 = 200 x 1.414 = 282.800 = 282.8 ohm Ans. Example 8.10. The space between the inner and outer conductors of an air dielectric coaxial cable of 100 ohm is filled with a dielectric whose & = 4. What is the characteristic impedance of the line now ? Ifa section of this line is used as a quarter wave transformer to match a load to the 100 ohms line, find the impedance of the load. What is the length of the transforming section at 100 Mc/s. (AMIETE, A & WP, Nov. 1966) Solution. The characteristic impedances of a coaxial cable having air dielectric and dielectric media of € are respectively given by % = 138 logio D/d .......air 2 = ¥38 togio D/d dielect = 4B tog D/d «.... jelectric € Let the characteristic impedance with € = 4 is Zy’ , — 138 Then 2" = FF loge D/d Zp’ = 69 logio D/d . Z _ 138 Dividing the two get, 2 Z ‘Thus the characteristic impedance on filling dielectric of € = 4 reduces to half of its previous value. By quarter-wave transformer formula 2 Zs = 100 Q; 2 = 2; % = 109 2 Th 2 5 rR _ (00)? toon Ze uy = 1002 Ans, Rea - | Length of transforming section at 100 MHz is given by Fig. 8.30 Quarter wave matching. TRANSMISSION LINES AND MATCHING NETWORKS 665 L 300 3 7m Ans wast as 4 100 * A=3m. ; Example 8.11. A coaxial cable having characteristic impedance of 50 ohm is to feed a half wave dipole antenna of 75 ohm. Design the necessary matching unit. (AMIETE, A & WP Nov. 1970) Solution. Given Z = 50Q; Zp = 75 Q; Zs = By definition 25 = B = 50% 50 _ 100 «335 ohms Ans __ Example 8.12. An antenna array presents an impedance of 300 ohms to the transmission line Seeding it. The transmission line consists of two open wire lines whose spacing is 9" apart and diameter of the wire is 0.1". Calculate the dimensions of a quarter wave line required for matching. Solution. The characteristic impedance of the feeder is given by Zp = 276 logio s/r where 5 = Spacing between two wires; = Radius of the wire. Given sa r= 0.1/2" = 0.05” = ss 900 _ 2 = 276 login 5.95 = 276 logio =z = 276 logio 180 log Z = 2.4409 + 0.4070 = 2.8479 or Zo = A’ log 2.8479 = 0.7046 x 10° = 704.6 ohms Ans. Characteristic impedance of the quarter wave line is now given by Z =VZs + Ze = V 7046 x 300 Given Ze = 300 Q ; Zs = 704.6 Q log Z = 2 [2.8479 + 2.4771] = 5) or Z = A’ log 2.6625 = 0.4592 x 10° = 459.2 ohm. Ans. Practically, the spacing of the quarter wave matching line would be the same as that of the main line. Hence if r be the radius of the line, then characteristic impedance Zp is written as 9 9_ % _ 4592 Zp = 276 logio = or log = = 396 = “376 9 2 9 Sear = 0. 10? = 45. a 2 = A’ logio 1.6641 = 0.4509 x O © r= ee or logio r = 0.9542 - 1.6641 or r= A’ log 1.2901 = 0.1950 or diameter = 2 x r = 0.3900 = 0.39” Ans. Example 8.13. It is needed to match a resistance of 200 ohm to a 300 ohm transmission line to reduce the S.W.R. along the line to 1. Determine the characteristic impedance of a suitable quarter wave transformer to be connected directly to the load. Solution. Since the condition S.W.R. = 1 applied along the main line, we require Zs, at the input to the 4/4 transformer, to be equal to the characteristic impedance Zp of the main line. Let Zy be the characteristic impedance of 4/4 transformer, then mL a or Zp’ = VZq Zs =¥300X 200 = 100 x V6 = 100 x 2.449 = 244.9 245 ohms. Ans. 666 ELECTROMAGNETIC FIELDS AND WAVES Example 8.14. An ideal loss-less transmission line of characteristic 60 ohms is connected to a resistive load. If the standing wave ratio on the line is 4, calculate (i) The value of the load impedance Zz, and (ii) The reflection coefficient at the load. (UPSC, CES, 1975) Solution. Given Z) = 602; SWR K+i1 06 1 16 A is A6 \_ 4.60% 4=240 ohm Ans, K-1 6-1 |~~©*| “04 Example 8.15. An ideal loss-less quarter wave (2/4) transmission line of characteristic impedance 60 ohms is terminated in a load impedance Z,,. Given the value of the input impedance of the line (i) When Z;, = 0, (ii) When Z, = =, (iii) When Z, = 60 ohms (UPSC, CES 1979) Solution. Since Z) = VZ5 Zi _B + Zp = Z.Given (i) Z,=0, (ii) Z.=0 * 7 (iii) Z, = 60 ohms and Zo = 60 ohms 2 @ Zs = #8 =e ie. open circuited Ans. 2 «i 25 = SOF = 0 ie, short circuited Ans. Zs = Gy = 60 ohms. Ans. Example 8.16. [fa loss-less transmission line of characteristic impedance 60 ohms is connected as shown in Fig. 8.31 (a) and (b), calculate the input impedance Zin of the line across the input terminals AB in each case. spe a 2my— eon | Ze? jroo 20. ¢ (a) (b) Fig. 8.31. Solution. .- Z = VZ,* Ze 2 BOF wo Ans. Z = 60.2 and Ze = 02 rR Example 8.17. Find out of the ratio of the conductor diameters for a coaxial quarter wave transformer for matching a source having an internal impedance of 300% ohm to a load of 20% ohm, if the characteristic impedance of a coaxial cable is 138 logio D/d, where D and d are the diameters of the outer conductor and inner conductor respectively. Solution. Since Zp = VZx + Zs or Zn (London Univ. Telecomn, 1951) ‘TRANSMISSION LINES AND MATCHING NETWORKS 667 where Zo = Characteristic Impedance of the A/4 line ; Zr = Load Impedance ; Zs = Input Impedance. Given Zp = 300% Q : Zs = 20% 2 . Z = ¥300 x 20 = 10 Y60 = 10 x 7.746= 77.46 ohms. or D/d = A’ logi 0.5611 = 0.3640 x 10 = 3.64, Ans. Example 8.18. Explain why it is necessary to match the impedance ofan antenna to that of the feeder and prove that such matching can be obtained with a quarter wave line. Ifa feeder having a characteristic impedance of 400 Q is to be connected to an antenna having an impedance of 70 ohm, calculate the spacing of a quarter wave parallel wire feeder to be inserted between the two if the diameter of the wire is 3 mn. Assume Zp = 276 log, S/r, where S = distance between centres of wires and r = radius of wire. (London Univ. Tele, 1944) Solution. From Fig. 8.32. Ms “ 2 = VR * Ze = VIO x70 = ¥280% 100 = 10 x 16.733 on. S=2Zo_ Z_=7Om Zo = 167.33 ohms. cr But Zp = 276 logio 8 Fig. 8.32. S _ 167.3 S_ xe - or logio > = “jog = 0.6061; = A’ log 0.6061 = 4.037 or S= 4,037 x 1.5 mm=6.2585mm~ 6mm Ans. Example 8.19. A dipole antenna whose input impedance is pure resistance of 100 ohms is to be matched at a frequency of 100 MHz by an open wire feeder having Zo = 600 ohms by means of short circuit stub. Assume feeder and stub without losses and having same characteristic impedance, _ (AMIETE, T/L Nov 1966, AMIE, Nov. 1972) Solution. Vide figure below, Given De Fig. 8.33. 2 ag l Bx tA! (04087) = x 22.2" =0.185m=185 cm Ans. 100 — 600 tan’ (—0.4898 ) NETIC FIELDS AND W/ 668 ELECTRUMAG! AVES wt Due to negative sign angle % 153.8" X75 obtained is subtracted from 180° 1.281 m= 128.1 cm. Ans. ‘Thus position of stub will be at 18.5 cm from the dipole antenna and length of stub will be 128.1 cm. Example 8.20. An open wire r. f: transmission line, which may be regarded as loss-free has a characteristic impedance Za of 600 ohm and is connected to a resistive load of 75 ohm. Find the position and length of a short-circuited stub, of the same construction as the line, which would enable the main length of the line to be correctly terminated at a frequency of 150 MHz. Solution, Given, Z) = 600 ohms; Ze = 75 ohms; f = 150 MHz faba? be ze tan! \2- x tar! V& - Hs tan! T = 7 am (xan ] = ox toa xs, = 222 0.108 m= 10.8.em Ans, ” =m 7 (HDR) a (-#230 2 (048 x (4158) x te + 180 22 = 158 he 2 = 0.8790 m = 87.9 cm = 88 cm. Ans. Example 8.21. An r.f line of characteristic impedance 600 ohms is terminated in an impedance 400+) 200 ohms. Calculate the voltage standing ratio. (AMIETE, A & WP Nov. 1965) Solution, Z = 600 ohms ; Ze = 400 + j 200 ohms; VSWR = ? = Zt=Zo _ 600-400-j200 200-7200 _ 100[2-j2] _2-j2 10-2 Ze+Z 600 +400+j200~ 1000+j200 100[ 10+j2}~ 10+j2* 10-j2 = (2 = j2.0) (10 ~ j2) _ 2x 10 - j2x 10-2xj24+74 1-7? 2 100 + 4 20 -j20-j4-4 16-jr4 16 x= Ma iW-j4-4_ 16 -jm_ 16 ; Ta Tos = oa ~4 jg 79-1538 - j x 0.230 or [K| = ¥(0.1538) + (0230) Z—tan? 2230. (0280) "0.1538 = ¥0.0236 + 0.0530 Z —tan"' 1.4954 = VO076 2 56.2°= 02768 2-562° Hieace vswr = LtIKI _ 1402768 _ 1.2768 1=|K|~ 1~ 02768 ~ 0.7232 ~ 1.7654 Ans. Example 8.22. When the far end of a finite transmission line is short circuited the impedance measured at the sending end is 4.61 Q resistive and when that is open circuited, the input becomes 1390 Q resistive. Calculate the characteristic impedance of the line. (Meerut Univ. M.Sc. Phy. Electronics, 1985) ‘TRANSMISSION LINES AND MATCHING NETWORKS 669 Solution, By eqn. 8.48 = V Zoe + Zee Given Zc = 4.61 Q; Zoc = 1390 2 = V461 x 1390 Q log Z = 4 [0.6637 + 3.1430] = 3.8087 _ 1.9033 Z = A’ log 1.9033 = 0.8004 x 10° = 80.04 ohm Ans. ixample 8.23. A transmission line of characteristic impedance 50 ohm is terminated by a tor 100 ohm. What will be the VSWR in the line ? Calculate the impedances at the voltage maximum and minimum position. (AMIETE Electromagnetic Fields and Waves, June 1988) Zr + Zp 100+ 50 Then VSWR= tt. 1414 =|K] "1-17 At voltage maximum iin the phase angle of the voltage reflection coefficient is zero and Solution. Since K hence |K|=|K| 20 or 2na= Zotyswrd=| Tet] 2 [ 118 |] 2=80%2=100 ohms Ans. Further, at voltage minimum, the phase angle of the voltage reflection coefficient is 180" and hence A=1Kl)2,-1,2,-0- Te “(eS ] = 5% Z => = 25 ohms Ans. Example 8.24. A 300 ohms transmission line is terminated in a load impedance of 100 + j 200 ohms. Calculate the voltage reflection coefficient. Solution, Given Zr = 100 + (200 2; % = 3002 x = 22a, 100 + j200 - 300 _ = 200 + ~ Zp + Zo 100 + j 200 + 300 ano 720 200(j-1) i 200 (7 + 2) _a(-1)-3j+2_3=3)_ su. cy, ~ -1-4 ~ =5 Zain = K= = 0.6 (1 -j) Ans. Example 8.25. Calculate the voltage standing ratio for isles line of characteristic impedance 50 ohms when it is terminated in 100 ohms resistive and 30 — j 50 ohms. nein . " Ze-% _ 100-50 _ 50 = Solution. = Se OE Ze = 1002 ; H=502 olution. K= 7,4 ZH ~ 100+ 50 ~ 150 ° 4 IKl=3 wow I LEL Les we | Next, when Ze = 30 - j50 670 ELECTROMAGNETIC FIELDS AND WAVES es -2- ~i5 , 8+j5 30 ~j 50-50 _ -20-j50 _ 10(~2-j5) _=2-/5 Then K=39"550450~ 80-)50 10(8-j5) 8-J5 on - 9 lB 40-10)- P x25 _-16+25-50j_9-j50_ 9 50; _ 4 igus 5617; mus 425 39 89 ao! otng . -1_ 95617 _ rapiggFOSISS Z— tax! | K | =¥ (01046) + (0.5617 > 2 tant FT 9 ge = VO.0109 +O3155 2 — tan” 5.369 = V0.3264 2 -79.5° = 0.5713 £-79.5° 1 + 0.5713 _ 15713 it K| _ 1+ 05713 _ 15713 _ 3 6652 = 3.66 Ans. Hence VSWR = 1K] = 1 - 05713 ~ 0.4287 Example 8.26. A ects parallel wire transmission line of characteristic impedance 200 ohms transmits power at a frequency of 100 MHz to a pure resistance load of 80. ohms. Calculate the position of attachment and the length of a short-circuited stub, of the same construction as the feeder, which will correctly terminate the main length of the line. (UPSC IES R and ME, 1971) Solution. Given Ze = 80.9; % = 2002 ; f=100MHz ; k=2;4=? _ ket a[ Ze _ 4 Nie b= Fe ta ve and b= i ta! FS = —300__ _ 300 _ Now = 77m) = 30073 ™ 3 3 2 = Sun Vou = 3 - 23° Soo ~3 qian NOS = 3° tan 0.6324— 5 x 32.3" x 23323969 _ sm radian = 180° = 7° FED og 70-5363 m= $3.63 em Ans. m radian = 180 . - 80X30 3 VT 3 -4 12.649 d he 1 = 1 = 1 = ‘an "30 = 200 2m * "T4992 XN! _ ye _3 sy — od t Bq * ar! (~ 1.0540) = 5 x (-465 d= ay x (+ 1335) x Fe = 15 x 1335 _ 200.25 Example 8.27. Calculate the position and length of a.short circuited stub designed to match 4200 ohm load to a transmission line whose characteristic impedance is 300 ohms. Solution. Given Z) = 300 2; Ze = 2002 ; L=hk=? = avi 14/2002 k tan Vi 309 = 3g tan’ V 0.6666 = A tan! 0.8164 Qn a 39.222 ~ is % 3922 x Fe = “SEe* = 0.10804 .m = 10.894 2 em Ans, te tan V2 2 gt V200%300 _ a 1 ¥60000_ A, 244.9489 ft Ze-Z 7s 200 — a tan "100-72 8 Tig sant 7 ; = fi tan (- 2.4495) 35 xO oxy nm 67.8° Dag oy Ue 22 = 122) = 03116 A m = 31.16 2 em Ans. ‘TRANSMISSION LINES AND MATCHING NETWORKS 671 Example 8.28. A half wave aerial tuned to 20 MHz is equivalent (between its feed points) to a non-inductive resistance of 100 ohms. It is to be fed by a parallel-wire feeder having a characteristic impedance of 500 ohms. Design a suitable transforming feeder between the main feeder and the aerial so that the former may be correctly terminated. (AMIETE Radio Communication System, May 1975) Solution. Given Zs = 5002; J = em | 00 2; =? Herel ta aeeeesea | la ANTENNA Zo = 223.60 Q Ans. Fig. 8.34, Example 8.29. A two wire line has a characteristic impedance of 300 ohms. It is used at 100 MHz. Find the minimum length required of a short circuited line that can be used to simulate an inductive reactance of 200 ohms. [Meerut Univ. M.Sc. Phy. (Final) Electronics, 1988] Solution. Given Zp = 200 Q; Z = 300 Q; f = 100MHz; |, = ? 300 Now A= 100 = 3m ‘The minimum length of short circuited line as given by Zp Zo _ 3,1 N200X300 __3_ Zp-Zy 20 200-300 20 tant 7 tan”! (—2.4495 ) -100 2 yo ey «112.2 x (-67.8°) = Qn x (+ 112.2") x 180 * 120.0 Example 8.30. At what distance from the aerial end of a transmission line of characteristic impedance 500 ohms should a stub be introduced on it so as to match the line with the aerial of 80 ohms input impedance, radiating wavelength being 100 metres. (M.Sc. Physics Electronics Meerut Univ, 1986) Solution. Z = 500 Q; Ze = 80Q; A = 100 metres; J, = ? vt af 2 = 100 pct af BO $0 ay 50 tat YT = ag 8 V 599 = gant VOTE = => tan! 0.4000 5x 218 _ 109 18 7 187 6.055 m Ans, Example 8.31. A loss-less transmission line of characteristic impedance equal to 50 Q, is terminated by an unknown impedance. On measurement, a VSWR of 3 anda shift in the position of minima of 2.5 cm are observed. The frequency of operation is 500 MHz. Determine the unknown impedance. . (AMIETE, A & WP Dec. 1975) Solution. Given » = 0.935 m Ans. 2n therefore Since 672 __ ELECTROMAGNETIC FIELDS AND WAVES The voltage minima is given by 2B ymin — 9 = 7 | ch=06m | __ 15.70 | or ar 2.61 radian = - 2.61 x 57.3" =— 149.553 degree= — 149.5° - jg R= _ -j2-61 _ Ze— 50 But K=|K [el =o = 05 ¢ am | a Zp -50 i or 05 [eos ( 149.5") ~ j sin (1495)] = 7°55 | "i 5 win ead oy) - Ze = 50 | or 05 [eos (90" + 595°) ~ j sin (90° + 59.5°)] = FA=ZF = 50 or 0.5 [-sin 59.5° - j cos 59.5] = a +50 } Ze - 50 | = 05 [0.8616 + j 0.5075] = aes = Ze | + [0.43080 + j 0.25375] = Dae or [0.43 + j 0.254] x (Zp + 50) = 50 - Ze or [0.43 Zp + j 0.254 Zp + 21.50 + j 12.200] = 50 - Ze | or [Ze (0.43 + 1) + j 0.254] = 50 - 21.50 -j 12.2 | or Zr (1.43 + j 0.254] = 28.50 - j 12.2 | =I 2 2 f'I" Nssoy aia |< tan ee Zp = 2850 12.2 a | Ze |= 28.50)” + (12.2 )' = 28.50 1.43 + j 0.254 (1.43)° + (0.254 y 1 0.254 Zian! 0254 | 1.43 1 1 a 14: 2 1.0864 s(54 | . 5 (2 x 14548 +2 x 171598241 9 5419 | ; [2 x 0.1533 + 2 x 1.4048] Zt T1162} 15581 | | A‘ log 2.5412 _ 347.7. _ mec | 2a = riog 15581 7 03615 % (08 Ze = 25412 ~ 1.5581 = 2.9831 or Zr = 961.8 ohm Ans. | Example 8.32, A 50 ohm transmission line is terminated by an unknown impedance. The VSWR is 4 and the first minima is formed at 2 cm from load end. The frequency of operation is 1000 MHz. Design a single stub line matching unit for the above conditions. (AMIETE, A and WP, June 1980) Solution. Given Z = 50 Q; VSWR = 4; stent f = 1000 MHz; Ze = 2; = 2; 1, pas TRANSMISSION LINES AND MATCHING NETWORKS 673 or or or oa or or or or and or. Now _ YSWR-1_4-1_3 tk 30 785 xlln_ =e = 9 ot = 073 x 3.14 @ = — 2.2922 rad = — 2.292 x 5.73" = ~ 131.3316" = - 131.3" 2p - Z 7 2-2 But K=|K[ee= 2% ya2m _ Ze - Zo IKI Ze + Zo or ote Zn+Z 06 [ cos (1313°) ~ jsin(1313")) = 2= 2 ATM nals .) _ Ze = 50 06 [-sin 41.3" - joos 413°} = 73 _ Za = 50 = 06 [066 + j 0.7513] = 7°55 _ 50 ~ Ze 0396 + 022578) = 75 (0.396 + j 0.2258) (Ze + 50) = 50 - Zr 0.396 Ze + j 0.2258 Ze + 19.800 + j 11.2900 = 50 - Ze Ze [0.396 + 1 + j 0.2258] = 50 ~ 19.8 + f11.29 Zp (1.396 + j 0.2258] = 30.2 + j 11.29 — 30.2 + 11.29 Zl= V¥(30.2 + (11.29 ye = F364 j 00058 % =|!" Yer396) + (02258) Zr = where x = V¥ (302) + (11.29) and y = ¥( 1396) + (0.2258 > y Now logx = ; [2 x 148 +2 x 1.0527] = 50684 = 25327 AY log 2.5327 = 0.3410 x 10° = 341 log y=5 512 x 0.1449 + 2 x 1.3537] 29972 - [4986 y= : log 1.4986 = 0.3145 341 low = 327 - Ty = os 0 25327 - 1.4986 ie = Al hag 3.0341 = 0.1081 x 10° = 1081 ohm. = ta! = oe = 93 _m _ 719 x 03 ty = 23 anv! VET = 5 tan 4.650 2x79 x 5 = _ 93 719 _719 = an = 1200" 12 X10? = 6.49 x 10 = 0.0649 m= 6.49 = 6.5 em Ans. ELECTROMAGNETIC FIELDS AND WAVES 03 9-1 V1081 X50 _ 03 14,, (et 1041 “on 1041 an = 03X63 63x3_ 6.3 _ 6.3 } 93 tant (0.2233) x10? = 1.05 x 107m 180 ~ 1800 600 6 = 0.105 m = 1.05 cm Ans. / Example 8,33. A lossless transmission line having a characteristic impedance of 75 ohms is t erminated in an unknown impedance Z,. The VSWR measured is 3.0. The nearest minimum from the load is found to be at 20 cm. Calculate Z, if the frequency is 150 MHz. (AMIETE Antenna and Wave Propagation, Dec. 1988) Solution. Given Zp = 75 ohms ; Zz = Load Impedance = ?; VSWR = 3 Ymin = 20 cm; f = 150 MHz; Then d= 20 = 2 VSWR-1_3-1 2 Hence K =321_2. VSWR+1 9 3+1° 4~— 05 The voltage minima is given by 2Byain-@ =m or 2x2Ex 20 gam or 2x28xt oan or 2B-o=n 2B nag ornl2-1]=9 o x|2=*]- -2]- 5 9 g-1]=@ o nl*32)=@ 0 x/-S]=0 or = @ = - 06m = ~ 06 x 3.14 = — 1.884 radian =— 1.884 x 57.3" =~ 107.95 degree or Z-% : Z- 15 B 7 J% = i(-1.e84) _ Zp = 75 ut Ka|k[ t= ZR or lsc as Jos] eve = 2-8 = B .. @ is in radian . 2-75 . Os 107: - K ir ae or 1 05 | Leos (107.95) ~ jsin(107.95)} = F+— 78 @is in degree 5 Zz - 75 os 90+ 17.95) 95) ]= or {0s (90+ 1795 )—jsin (90+ 17.95 )] = Z2— 7 ig | ~ 4-75 or 0.5 [- sin 1795 - jeos 17.95} = = . . 4-15 = 05 [sin 17.95 + joos 17.95] = 24= 75 [sin Joos 17.95 | = A= , Za - 75 = 05 [0.3084 + j 0.9520] = t 409520) = 7 75 . Z- 175 ~ (0.1542 + 70.476) = = 75 t J0476) = 775 or + (Z +75) [ 0.1542 + j 0476] = 75 - Z or Z, (0.1542) + j0.476 Z + 11.565 + j 35:7 = 75 - % or Z (1 + 0.1542 + j0.476] = 75 ~ 11.565 - 535.7 or Z (1.1542 + 0.476] = 63.435 ~ 535.7 7, = $3435 = j 357 1.1542 + j0.476 TRANSMISSION LINES AND MATCHING NETWORKS 675 V(6B343 7+ (35.7) _VAOISIOF ITED _ VSDTAS _ 72.7863 || *Vcrisys costes? “"Y1.3225+ 02265 ~ 0.5490 ~ 0.7409 ~ 9824 ohms Ans. Example 8.34. A 50 ohm lossless line is terminated in an unknown load. The distance between adjacent voltage minima is d= 8 cm, the VSWR is equal to 2, and the first voltage minimum is situated fi = 1.57 cm from the load. What is the value of the load impedance Z;, ? (AMIETE A and WP, Dec. 1989 Old Scheme) Solution. Given Z) = SO ohms; yin = 1.57 em; VSWR = 2 The distance between adjacent voltage minima d = 8 cm which will also be equal to the half of the wavelength (ie. 4/2) Hence d= F=8cm= m or A/2 = 0.08 metres or A = 0.16 metres Now The voltage minima is then given by 2B ymin -@ = or 2x 22 x yin — 9 = Qn 157 2x2mx 157 9 mx 157 _ or 2X Og X og OER oF 4 Q=% oF 5 1.57 =n ('57=4)_ (2248) 0 ce een ut -1)== (44 =n (=42) - 2 (-0.6075) or @ = — 1.9075 radian = - 1.9075 x 57.3 = — 109.30° Z - 50 now Ka | Kl = 750 0.33 e/(-96075) _ 4-3 Zi - 50 0.33 [eas (109.30) ~ j sin ( 109:30)] = 7-35 Z - 50 0.33 [cos (90 + 19.30) - j sin (90 + 1930)] = z a0 . Z - 50 or 0.33 [~ sin 19.30 ~ jeos 19.30] = 7-20 ; — 50 or — 0.33 [0.3305 + j 0.9438] = z +50 , _0-% 0.1090 + j 0.3114 = +50 or (Zz + 50) (0.1090 + 70.3114) = 50 ~ Zz, or Z (1 + 0.1090 + j 03114] = 50 - 5.45 — j15.57 or Z (1.1090 + j0.3114} = 44.55 — 515.57 or 7, = 455_= j 1557 1.1090 + 70.314 6/6 ELECTROMAGNETIC FIELDS AND WAVES a CCEDENEDY _ NI98470-+ 242.42 _ 22771244 _ 47.1924 , || N10 + (031) Vi21+00961 Vi3061 ‘1.1428 =41.2954= 41.3 ohms Ans. Example 8.35. A lossless line in air having a characteristic impedance of 300 ohms is terminated by an unknown impedance. The first voltage minimum is located at 15 ¢m from the load. The standing wave ratio is 3.3. Calculate the frequency and terminated impedance (Bangalore Univ. Electromagnetic Fields and Waves Oct. 1983) Solution. Given VSWR = 3.3; Z = 300 ohms; ymin = 15 cm = 0.15 m Zp = ? = load impedance = Z,; f=? VSWR -1 _ 33-1 _ 23 , = VSWR=1 33-1 23 _ g5s4g Since K=Vswe +1 3341 43 Zr- Jee Now [k| el? = Bo o _ Zr- 300 @=0; e=1 [Kl] e® = 7300 _ 2x = 300 or 0.5348 x 1 = 7 a09 or 05348 [Zp + 300] = Ze - 300 0.5348 Zp + 300 x 0.5348 = Zz — 300 Zr [0.5348 — 1] = - 300 - 160.44 Zr [ — 0.4652] = — 460.44 or Zr=+ Now the voltage minima is given by 2 B Ymin - 9 =] But reflection coefficient K is not having phase angle @ i.e. @ = Oso that oa: Qn. 1S _ 1s _ 2Bywin ="; 2-5 Type mor heap 300 300 = nce ==) = 6 MHz = 500 MHz Ans. Example 8.36. The voltage minimum occurs 18 cms from the termination of a lossless line of “Length 52 cms. Adjacent voltage minima are 20 cm apart. The voltage standing wave ratio is 2.5 and Nels icteristic impedance is 300 ohms resistive. Using Smith chart or otherwise, find the input impedance the load impedance of the line. (Bangalore Univ. B.E Electromagnetic Fields and Waves, Oct. 1983 ) . L WY Solution. Given yin = 180m = 75 m; 1 = 52 om = 3m; vewR = 25 Z = 300 ohms Voltage minima occurs atd = A/2 = 20 cm z A= 40em =H m= 04m Zn = 25 Zr=? VSWR-1_ 25-1 _ 15 ~ K=\Vowr ol 725 1 3g = 04285 TRANSMISSION LINES AND MATCHING NETWORKS 677 The voltage minimais given by 2Bynin-g=m or 2:48.18 gan 1 2m 18 4x °F 2 og "gg OF ee or g=18n-m=+08 n= + 2512 rad or @ = 2512 x 573 degree = 143.9376" For, load impedance |K| e/@ = 22— Ze + Zo 0.4285 2512 _ Ze - 300 By Euler’s Theorem Se Zr + 300 e!® = cos @ + j sin® so that Ze - 300 1.47 ———. 0.4285 [cos (143.93) + j sin (143.93)] = FS ; _ Ze - 300 0.4285 [cos (90 + 53.93) + jsin (90 + 5393)] = 7399 . _ Ze ~ 300 or 0.4285 [~sin 53.93 + jeos 5393 | = 7" 1305 _ Ze = 300 or 0.4285 [ - 0.8085 + j 0.5885] = 7°59 or [-0.3464 + j 0.2521] [Ze + 300] = Ze — 300 0.3464 Ze + j 0.2521 Ze + j 75.63 — 103.92 = Zz — 300 or — Ze { 1 + 0.3464 - j 0.2521) 03.92 — 300 - j 75.63 or = Ze [ 1.3464 - j 0.2521] = - 19.608 - j 75.63 _ 196.08 +j 75.63 or ® = 70.2521 — 1.3464 we | C9608 F + CSS) _ VIRHTSG + S79 ¥38447.366 + 5719.8969 = | e| = (0.2521)? + (13464)? -V0.0635 + 18127 V4G167262 _ 210.16 2 = 210.16 _ 153.43 ohms Ans. |2e| = “Visrer > 13607 ohms Ans, ‘The input impedance Zi, in terms of reflection coefficient K is given by =2i81 J = 280) Kj e* ei] 7 U4 [Kl e! es ] [1 = [K| el(@= 2804 For this, let us calculate (9 — - vr ) first Zin = ZH Now o - 261 = 2512-2 x 2F x 052 2x 2x 3ld x 052 6. = siz ~ RENT ANS? = 2512 ~ $3312 - 2519 — 16328 9 - 2B = — 13.816 rad | @ - 2B/| = + 13.816 rad = + 13.816 x 57.3 degrees = + 791.6568" Now @ — 21 = 791.65" = 791.65" ~ 360° x 2 = 791.65" — 720° = 71.65" 678 ELECTROMAGNETIC FIELDS AND WAVES LEK eO-28O) re Kel [14 k(c0s 71.65 +) sin 71.65")] Then Zu = 20 city * [1K e385 [1k (cos 71.65 + sin 71.65°)] 4285 (0.3142 + j 0.9494 [1 + 0.1346 + j 0.4068] Zn = Zp LEA 0428S ( i Yen ~ [1 - 0.4285 (0.3142 + j 0.9494) ] [1 - (0.1346 + j 0.4068 )] Zn = Zo 1.1346 + j 0.4068 =h (1.1346 + [0.4068 } [0.86 +j 0.40 © [0.8654 — j 0.4068 } [(0.86) + (0.40)"] =% [1.13 + j 0.40 0.86 + j 0.40 0.7396 + 0.16 =Z 0.9718 + j 0.344 + 70.452 - 0.16) _ X 0.8118 + j 0.796) - 0.8996 * 0.8996 = 300 (0.9024 + j 0.8848) = (270.72 + j 265.45) or | Zin | = V(Q70.72)" + (265.45 = V73830.758 + 70463.702 = V144294.46 = 379.86 ohms Zin = 380 ohms Ans, Example 8.37. A 300 ohms line feeding an antenna, has a standing wave ratio of 4.0 and the distance from the load of the first voltage minima is 28 cm. If the frequency is 150 MHz, design a single stub matching system to eliminate standing wave ratio from the maximum possible length of the line. (Bangalore Univ. B.E. E.F.W. Nov. 1984) Solution. Given Z)=3002; VSWR=4.0; ynin=28em ; f=1S0MHz; L=?; 4=? = 300 _ Now = eon The first voltage minima is given by 2 ymin - = 1+ 2-2%- ynin— =H =2-28. -n= 2m, 28 or g=2 7% Ymin marx 100 ~ =n 8-1 )on( 4-1 )on (4225) og (1 wo(Bs)eo( S82) e (pasa @ = ~ 1.3816 rad =~ 1.3816 %57.3"=-79.16" e Lrad = 57.3" _ YSWR'-1_ 4 3] Now K=Vswre +17 441757 °° ° The value /, (position of stub from the load end } is given by eqn. 8.130 Xd 2 b= 7 (@ +m cos? |K]) m and length of stub I, is given by eq b= * tant VE=-1KT Putting the values we have 2 a L = 75 (@ + 0 - cost | KI) Since 679 TRANSMISSION LINES AND MATCHING NETWORKS 2 aj 1 . Hence y= 7 (-0.44 m+ 0 — cos [0.6 |) = 5 = 10.56% - 531°] a 53.1 x *] Qn = [056m — 0.295 m] “on 180 x [0.265 m] = 0.1325 m = 13.25 cm Ans. t= naman! DESLKE 2 2 tage Vi =(6F | 2(K| 3% 20” 2x06 [oss n- and = 0.18694 m = 18.74 em Ans. Example 8.38. A 300 line is terminated in a load Ze such that S is measured as 4.48 and the distance of location of the first voltage minima from the load is 6 cm. Explain how this can be matched to the load with a single shunt short circuited stub. Find the stub length and the distance in cm, if f= 200 MHz. ; (AMIETE, Principle of Comm. Engg. 1981, M.S. Univ. Baroda, April 1985) Solution. Zo=300Q; VSWR=4.48; ynin= 6cm; f=200MHz ; L=?; b=? VSWR - 1 3.48 300 = EWR 1 ata = ; aeMeis K VSWR +1 448 +1 548 og 5 200 1 The first voltage minima given by 2B ynin-Q = of 9 AX TEXT -@ on 6-375 ]_ o 75°"? . [ 375 |- ° or ° ~ 0.84 x 3.14 = — 2.6376 x 57.3" = - 151.13" a = cos =15 — cos 0.6} = 12 _ ssa Fy (+ ~ c08 [K]) 0.84 n +n — cos 06] = 47 [0.16 m - 53.1°] 1s 531xm]_ 15 _ _ 1S 0.2025 -2 [ots =~ 5 ]- 23 tse 0.295 n] = - 0.135 x =P = — O28 = ~ 0.050625 m = - 0.506 cm |4| = 5.06 cm Ans. 1-2 [1 aN “HE. i3/ 1 VI -]0635F ] 15 | tan! Wi — 0.4032 L, = ny = tan’ = Qn 21K] 2n 2|K] Qn 2 x 0.635 1 105968 5 536 _ 15 = a = ri att 1 a [ws a In 8 F97 = Ze ta! 0.3768 - + 30.945 _ , = gig % 2063" X 785 = “agy = 008595 m= 85cm Ans. Example 8.39. Consider a line of 300 Q termi oss ne, Casal of terminated by an unknown impedance. The standing 200 Mit. Find the terminating impo ae Situated from the termination, when the frequency is the wavelength is 150 cm. ry the use of Smith chart assuming that tine is in air and that », Summer 1965 ) Solution. Vide above example K = 0.635 ELECTROMAGNETIC FIELDS AND WAVES 680 @ = — 2.6376 radians = - 151.13 degrees Ze — 300 ~ 26316 1 _ Zr = 300 ‘Then IK]! = 709 0.635 [e~! 1= 3 + 300 oo Zp - 300 0.635 [ cos ( 151.13) — jsin (151.13) ] = Zp + 300 Ze — 300 0.635 [cos (90 + 61.13) - j sin (90 + 61.13)] = 5+ 300 . Ze - 300 0.635 [~ sin 61.13 ~ j cos 61.13] = F595 — 0.635 [0.8759 + j 0.4825 ] [Ze + 300] = Ze ~ 300 + [0.5561 + j0.3063] [Ze + 300] = 300 - Ze Zp (1.5561 + [0.3063] + 166.83 + j 91.89 = 300 Ze (1.5561 + j 0.3063 ] = 300 — 166.83 — j 91.89 Zp ( 1.556 + j0.3063 ] = 133.17 — j 91.89 Z_ = 3317 = j 91.89 B® "1,556 + j 0.3063 or or or Rationalizing, we have 2q = (SBT = j 91.89) (1.556 = 0.3063) _ 207.21 - j 142.98 ~ j 40.78 - 28.14 (1.556 + (0.3063)? 2.4211 + 0.0938 = 179.07 ~ j 183.76 _ — Zp 2514 71.2036 - j 73.0685 Ans. or | Ze | = 150699526 + 5339.0056 = {10408958 or Ze = 102.02 ohms Ans. Example 8.40. An U.HLF. lossless transmission line working at 1 GHz is connected to an unmatched load producing a voltage reflection coefficient of 0.5 Z 30°. Calculate, after deriving necessary relations, the length and the position of a single stub to match the line. (AMIETE Transmission Lines and Networks, May 1975) Solution. Given that f= 1 GHz; K=05; 9 = 30°; =| =? = 300 _ : = = 39° = 30x" Now 2 = io95 = 03m i K=05 and @ = 30" = 0% =A cos" K|p =| B A Alfa % Hence, k= 7 19+ cos [ein gyl §rn-oor 105] |=] 242-60 aA Ry neon JAP im mn] af7m 20) 2% sn alee 389 |- 74 6 $A rece vical .0625 m= 6.25 em Ans, Now b= getan! SS LAE tet MOS gg-1 ROIS 7 2K] 2x05 a 4k ok = Jog 0.8660 = 3° x 40.9" = Ex 40.9 x = 0.034083 m=3.4083= 3.41 cm Ans. 2 m _03 40.9xn_ 12.27 180" 2n™* 180 ~ 360 681 TRANSMISSION LINES AND MATCHING NETWORKS Example 8.41. The parameter of a cable are : Resistance = 65 ohms/km ; Inductance = 1.6 mH/km; Capacitance = 0.1 F/km; Conductance = 2.25 1 Q/km. Calculate the characteristic impedance. Solution, Frequency is not given. So let us assume © = 5000 rad./seconds Hence Z = R + jwL= 65 + 5000 x 16 x 107 = 65 + j 80 2 ant & [Z| =V (657+ (8.0) Z tan 0.1230 = V4225 + 64 27° = V4289 27" = 65.49 27° Now ¥=G+joC=2.25 x 10% +) 5000 x 0.1 x 10% = 10° [2.25 +500} Z tan SR | ¥ | = 10% V5.0625 + 250000 Z tan"! 222.2= 10° ¥250005.06 7 89.7° | ¥ | = 10% x 500.0505 2 89.7° Zz 65.49 ZT" ———5 y 1-89.7 Z. = 0.130978 x10" Z A ¥ 500.00505 x 10° 2 89.7° 0878 x 2 = VO1B007E IT" ZZ - 9.361900 x 10? 2 -SBT - 361.909 2-41.38" Ans. Example 8.42. A uniform line has the following primary coefficients at @ = 5000 radians/sec. L=1 mH/mile, R = 40 ohm/mile, ; C= 0.1 MF/mile, G = 1 Mhos/mile Calculate its characteristic impedance. (AMIETE, Principle of Comm. Engg.. Dec. 1981) Solution. Z=R+j@L=40+j5000x1x 10° =404j5 [2|=V OF SY 2 tan 5 = 160025 Lean 0.125 = V1625.00 27.11 °=40.31 27.11" . ; 5 196 ; 1 500 =G+joC = 1x 10% +7 5000 x 0.1 x 10%= 10° [1 + j 500] 4 tan 32,000 \ This suggest that value of B should be chosen in such a way which should have, the value | approximately B = 0.2355. Hence, put = 2, 3, 4,5, 6,7 etc. till B approximates to the given values. Let us try B=6 0.002268 + 0.2355 = 0.2332 Ans 2355. Once the correct value of B is = — 0.002268 + 0.03925 x 6 = This is in close proximity with the given value of B known, we may also calculate primary constants from eqns. R+joL=Px Zand G+joc=F Example 8.58. A telephone cable of 60 Km. long has a resistance of 10 ohm/km and a. capacitance of 0.008 . F/km. Calculate the wavelength of the line. (Bangalore Univ. B.E. Ill Supplimentary Fields, Lines and Waves 1985) R Solution. In case of telephone cables a = B = V eer Let us assume here the standard frequency f= 1600 Hz as the same is not given in the problem. The length of the cable / = 60 km. Hence Total resistance of telephone cable R = 10 chm/km x 60 km = 600 ohms and total capacitance of telephone cable C = 0.008 x 10° F/km x 60km = 0.48. 10°F @ = 2nf = 6.28 x 1600 = 10048 rad/s | af ORE _ [10048 x 600 x 0.48 x 10% 2 2 = ¥3014400 x 0.48 x 10% = V1.446912 = 1.2028 rad/m 7 =. m= 52211 m Ans. Example 8.59. The constants per km of a certain cable are R= 42.9 ohms; L=0.7 mH; C=O01 UF; G = 24 p mhos Calculate the attenuation function, characteristic impedance, and the phase velocity when = 5000 radians/second. (Bangalore Univ. B.E. Fields, Lines and Waves Oct. 1983) | Solution. Z=R + jw L=42.9 + jx 5000x0.7 x 107 =42.9+j3.5 = VG29)+ BS) Lean 35 _ \igaOAT + TDS Ztan" 0.0815 429 = V1852.66 24.63° [Z| = 43.005 2 463° Y=G+ joc = 24 x 10° + j 5000 x 0.1 x 10% = 10% x [24 + j 500] | vy) = 10% V(24y" + (50077 tant 20 10° ¥5.76 + 250000 2 tan” 208,33 = 10 ¥250005.76 2 89.7" | ¥ | = 10% x $00.00575 2 89.7° Therefore, B Hence A= TRANSMISSION LINES AND MATCHING NETWORKS: 689 =VZ-¥ = V43. 4 Hence P=VZ-¥ = V43.0425 x 10x 500,00875 Z 463° + 89.7" _ sai ag7 x10 Via 2 = 146, s y= P = 146.70 x 10° < 47.16" = 0.14670 Z 47.16 = 0.1467 [ cos 47.16 + j sin 47.16] P = 0.1467 [ 0.5996 + j 0.7335] = 0.0879 + j 0.1076 = a + jB ct = 0.0879 nepers/km Ans, and 6 = 0.1076 rad/km Ans. or = @ _ 5000 _ ¥ = B = 0.1076 = 4646840 = 0.464684 x 10° km/s Ans. Z_ 43.0425 2 4.63" F md De aA r = Vaasa cio 24:63" - 89.7 Y 500.00875 x 10° 2 89.7 ~ » 086084 x 10 2 = 85.07" = 0.29340 x 10° Z Sar = 293.4 Z - 42.53° Ans. Example 8.60. Calculate the characteristic impedance of a coaxial line at 100 MHz when the primary constants of the line are 0.098 ohm/m, 1.5 x 10° mho/m, 0.32 H/m and 3.45 PF/m. (AMIETE Principles of Comm. Engg., June 1984) Solution. @ = 6.28 x 100 x 10° = 628 x 10° Z=R+ jOL = 0.098 + j 628 x 10° x 0.32 x 10° = 0.098 + j 200.96 Z| = VCOOEST DOODEY Z tune? 200.96 or |Z] ( YF + (200.96) 2 tant! Tose 0.009216 + 4038492 2 tan”! 2093.3 = 140384929 2 90° | Z| = 200.96 2 90° Y¥=G+j@C=15 x 10% +7 628 x 10° x 34.5 x 10? = 15 x 10° + j 628 x 34.5 x 10% = 10°[ 1.5 +/21666] = 10% [ 0.015 + j 216.6] Z tan cue |v |= 10+ [ ¥0.000225 + 46915.56 ] 2 90° = 10 [ Y46915.562 ] 290" = 216.6 x 10% £90" _ | ¥ | = 0.0216 2 90° a[Z 200.96 290 _ asa570 -4/Z = = 70 2 = Y 0.0216 290° a8 ° Example 8.61. A uniform line has the following primary coefficients at «= 5000 radians/sec. L=1mH/mile, R= 40 ohms /mile, C= 0.1 p.f/mile G = 1 wmhos / mile. Calculate its chasacteristic impedance. (AMIETE Principle of Comm. Engg,, Dec. 1981) Solution. w = 5000 1/s Z=R+jol = 40 +j 5000 x 1x 10? = 4045 [zj=V 40" +5) Ztan! - =V1600+ 25 Z tan! 0.125 = 40.31 27.11" 1 x 10% + j 5000 x O.1 x 10% = 10% [ 1 +500] 96.45 20° ohms Ans. Y¥=G+joC= IYl= 10° Vi2 + 600)" Zant 209-10 SV + 250000 290" = 10° x 500.00 2 90° - - 90 Hence z=) 2 = wa ZT _ Yop ca Z HE ¥ 500 x 10° 290° 2 = 0.2839 x 10° 2-22 283.9 (ZL-A1A4S" = 284 2-414" ohms Ans. 690 ELECTROMAGNETIC’ FIELDS AND WAVES, Example 8.62. Calculate the characteristic impedance and propagation constant ¢ a transmission line with the following constants : R= 2 ohms/m; 02 mos’; L=1mH/m; C = 3 wpF/m. . May 1976) Solution. Since frequency is not given so let us assume w = 2mf = 5000 rad/sec Z=R+jol=2+j 5000x1x 10% =2+55 [2|=N2 45? Z tan £ = ¥20 Z tan” 2.5° = 5.3851 2 68.2” ohms/m Again, = Y=G+joCc Normally the value of G is in micro mhos/metre and hence. Let us put G = 02 x 10% mhos/m. ‘Then ¥=0.2 x 10% +j 5000 x3 x 10? = 10 [0.2 +j 15x 107] = 10% [0.2 + 0.015} | ¥ | = 10° Y0.04 + 0.000225 7 tan? ais = 10 0.040205 Z tan”! 0.075 [¥| = 10° x 0,2005617 2 4.3° = 0.200861 x 10% 24.3" aafZ _4 [53851 2682" _ [5.3851 x 10° _/ 68.2" - 4.30" Hence 20 Wy = Vo200561 x 10% 24.5" 0.200561 2 =¥26.850 Z a8 = 5.18 Z 31.95" Ans. Now P=VZxY = V5.3851 Z 68.2° x 0.20056 x 10% x Z 43° . es = V1.0800356 x 10° Z son = 10392476 x 10° Z Bs = 0.001039 4 36.25° = 0.001039 [ cos 36.25° + j sin 36.25° | 0.001039 [0.8079 + 0.5918] = 0.0008395 + j 0.0006148 = a + iB Hence & = 0.0008395 nepers/m Ans. B = 0.0006148 rad/m Ans. Example 8.63. An open wire telephone line has R = 10 ohms/mile ; L = 0.0037 Henry/mile C= 0.0083 x 10 Farad/mile ; G=0.4 x 10% mhos/mile. Determine its characteristic impedance attenuation and phase constants at 1000 Hz. (AMIETE Transmission Lines and ‘Networks, Nov. 1969, Dec. 1973) Solution. w = 6.28 x 1000 = 6280 ‘Then series impedance is Z=R+joL = 10 +j x 6280 x 0.0037 = 10 + j 23.236 IZ|= VOY + (23.236 2 tan! 23236 = V1004 539.9116 Z tan 2.3236 = V639.9916 Z 66.71° = 25.2964 2 66.71° Now Y= G + j@C = 04 x 10% + 6280 x 0.0083 x 10%= 10° [0.4 + j $2.124] [¥| = 10° YOIe + 27169113 7 tan Slee = 10° V2717.0713 Z tan 130.31 [¥ | = 10° x 52.1255 + g9.6° TRANSMISSION LINES AND MATCHING NETWORKS 691 _afZ_ 25.2964 266.11" _ Saag xqgrt / 6:11 = 99.6" Hence Z Y Bssxio? coe eee z - = 0.6966 x 10° Z =e = 0.6966 x 10°? 2-11.44" Zo = 0.6966 x 10°? [cos 11.44" - j sin 11.44°] = 696.6 [0.9805 - j 0.1988] Zo = 683 — j 138.48 ohms Ans. P=VZXx ¥ = V25.2966 266.71 x 52.1255 x 10% Z 89.6" = ¥1318.5979 x 10° Z 6 + 6671 = ae 0 tot 15692 = 0.03631 [cos 78.16 + j sin 78.16] = 0.03631 [0.2076 + j 0.9788] a + j B = 0.0075379 + 0.0355402 Hence a = 0,0075379 neper/miles and = 0.0355402 rad/miles Ans. Example 8.64. Measurements on a terminated transmission line gave the following results. VSWR = 3.2. Location of the first voltage minimum = 0.23 2. from the load. Characteristic impedance = 50 ohm. Calculate the terminating impedance. (AMIETE Principle of Comm. Engg., Dec. 1984) Solution. The terminating impedance is given by aa~%[ 1 _ ra 5 —jtan B dy Given VSWR=5=32; B=2%, dy=0.232 B dm = 2s x 0.23 2 = 0.46 m = 1.444 radians = 1.444 x 57.3° = 82.76" 7 2a=20|" spesauns226 | so[ 1-j32 ae | 124.9798 3.2 —j tan 82.76" 3.2 —j x 7.8062 3.2 — j 7.8062 Vis (24.58 1 = 24.98 Ze=50 Ve (24oy 20 |. of 1+ 624.00 Z tan! - 24.98 ~ 2 2 1 78 10.24 + 60.84 Z tan”! — 2.4375 V2) + (78)? Zan ~ 35 }-s01 29685 2-201" Zp = 148.275 Z - 20.1° ohms Ans. Example 8.65. The input impedance of a cable, 10 miles long, is 2930 Z 26° ohms with the far end on open circuit and 260° Z - 32° ohms with the far end on short circuits, the frequency being 796 Hz, calculate the ‘primary constants of the line. (London Univ. (Telecomm.) 1949) Solution. w = 27f = 6.28 x 796 = 4998.88 radians/sec. -32 Now ZH =VZoax Ese = VI930 226° X 200 232" = 12930 x 260 LBB = V761800 Z 3 = 872.8115 Z -3° 692 exevinwmmuniine tune seme tnwey 260 Z — 32° 32° 32° -26° = -58° Further, tanh P= vie Bec = vous L275 = 029788 2 = 2930 226" = 0.29788 Z - 29° = 0.2978 [cos 29° —j sin 29° ] = 0.2978 [0.8746 —j 0.4848) tanh PI = [0.2604 — j 0.1443 ] -e" _ 0.26 ~ j0.14 Wy gl 1 Applying componendo and dividendo, we have ; Pl eae Mt ete en" 026-jOl+1 5 2e _ 1,26 — j0.14 Ay el Ply e-Pl 1 - 0.26 + j0.l4 rae 0.74 + j0.14 2 N26) + (0.14)? Z tan" 0.1471.26 _¥ 1.5876 + 0.0196 Z tan '—O.1111 °r eV (074 + (0.14) Z tan 014/074 V0.5476 + 0.0196 Z tan 0.1891 apt _ V16072 Z— 632 _ 1.2677 26.32" - 10.7" _ 4 6g53 78.51" é V¥0.5672 210.7" ~ 0.7531 2 +6853 Now — loger 2 O= loge r-e/(9*?"") = log. r+ jO7?"™ loge e=loger+j (0+2nT) or where loge e = 1 The term 2% has been introduced as the vector e/(° *7"*) is same forall values ofm but the logarithm of the vector depends on the value of n. Now taking log of both sides of above eqn., we have A 2 PI loge e = loge 1.6833 + j (-8.51° + 2nm) or P= 3 [loge 1.6833 + j {(-851°) + 2nm}] Since 1 = 10 miles 53.47 nd 851° = & _ 234t a eo. = 0.2970 rad. 1 Hence To [ 2:303 logic 1.6833 + j {- 0.2970 + 6.282} ] “2x 1 39 (2:303 x 0.2261 + j {= 0.2970 + 6.28n}] P = 0.026035 + j (- 0.01485 + 0.314n) =a +jB Hence & = 0.026035 ; B= (0.01485 + 0.314n) Since Tf = 6.28 x 796 = 49998.88 0.01485 + 0.314 n For 0.01485 + 0.314 = + 0.29915 For 0.01485 + 0.314 x 2 = - 0.01485 + 0.628 = + 0.64285 Since vp = & = 4998.88 'e BR ca = 16710.279 miles/s ++ B = 0.29915 0.16710 x 10° miles/s 693 TRANSMISSION LINES AND MATCHING NETWORKS _ Now calculating the velocity of propagation by putting the value of B = 0.64285 4998.88 *» * 0.64285 It is evident from the above two values of v, that last value is improbably low, as such we may select B = 0.29915 as the velocity of propagation is also not given in the problem so that we can cross check it. Hence B = 0.29915 1 0.2991. and accordingly P = 0.026035 + j 0.29915 = V¥(0.0260)" + (0.2991 )° 2 tan’ 0.0260 = ¥0.0006 + 0.0894 tan’ 11.50 = 10.0900 2.85.8" = 0.3001 285.05" In order to calculate R and L we may use R+j@L=PXZ=0.3001 Z 85.05" x 872.8115 2-3" = 261.93 2 85.05" - 3 = 261.93 2 82.05° = 7776.1219 miles/sec = 0.0777612 x 10° miles / sec. R+j @L=261.93 [ cos 82.05" + j sin 82.05° ] = 261.93 [ 0.1383 + 0.9904 ] = [ 36.22 + j 259.41] Hence on equating real and imaginary, we get R = 36.22 ohms/miles Ans. and OL = 25941 259.41 _ _ L = Joog.ag = 9.0518947 Henry = 51.89 mH Ans. Further, for calculating G and C'we may use G+joc= 0.0003438 2 85.05° + 3° = 0,0003438 [cos 88.05" + j sin 88.05" } = 0,0003438 [ 0.0341 + j0.0994] = 343.8 x 10° [ 0.0341 + {0.0991 ] = 11.72358 x 10° + j 34.17372 Equating real and imaginary G = 11.723 x 10°° mhos/miles = 34.17372 x 10 mhos/miles Ans. and @ C = 34.17372 or c= 34.17372 x 10° = 0.068362 x 10° Farad = 0.068362 uF/miles Ans. 4998.88 Example 8.66. Impedance measurement ona 500 km length of cable at © = 10,000 rad/sec under open circuited and short circuited conditions gave the following results : Zoc = 2000 Z-80° ohm; Zsc = 20 Z 20° ohm Calculate the values of Zp, 0, B. (AMIETE Trans. Lines and Networks, May 1976, B.E. Bang. Univ. 1985) Solution, Z» = VZoexZsc = V2000 Z~ 80" X20XZ 20" = Van000 Z = 80+ 20" = 200 £ =2- 200 2 - 30° ohms Ans. Nowtanh pr = af 28 _ [202200 _ [1 Lew 1p so Zoc 2000 Z- 80° 100 10 % = 0.1 [cos 50° + j sin 50°] = 0.1 [0.6428 + j 0.7660] e =e" _ 0.06428 + j 0.0766 el A en + en I ELECTROMAGNETIC FIELDS AND WAVvEg 694 Applying Componendo and Dividendo Pe Meet eM 1.0642 +j 0.0766 o 2 ef 1.0642 +] 0.0766 2 = 7 ~ T— 0.0642 -j 0.0766 2e-P! 0.9357 -j 0.0766 eee Mie 4 em A106 FT OOTEEE LZ tan! 9.0766 ] . (1.0642 )? + ( 0.0766 )* Z tant" Oey __V11335 4010058 Z tan” 0.0719 2a or eM = = 0.8755 + 0.0 = tan 0.0818 V(09357 Pe ODTGS YZ ~ tan! 9.0766 ~ ¥0.8755 + 0.0058 Z— tan (0.9357 + (0.0766)* Z—tan"' Doser V1.13832_Z tan” 0.0719 1.1365 24.1" 44.7" = 1.1365 28.8" 0.8813 2 -tan' 0.0818 Taking log of each sides, we have 2PI = loge 1.1365 + j (88° + 2nm) or P= (2308 x opie 1.1365 + (88° +2n)] a j (0.1535 + 6.28 P= srg (2308 x 0.0554 + j (0 n)] 88xn__ 88x3.14 27.632 + = BBX pag = 88X3.14 _ 27.632 _ 9 d 1=500m=05 ki Since 8.8° 180 rad 180 180 0.1535 radi an m cm or P zg louzs + j (0.1535 + 6.28 n) }=[ 0.1275 +j( 0.1535 +6.28n)]=a+jB Hence, & = 0.1275 neper/km and =~ = (0.1535 + 6.28 m) radians/km 0, B = 0.1535 radians/km Ans. 6.4075. rad/km 65146.57 = 0.65146 x 10° km/s Now, when on When n Since %» . and with next value ofB vp = i = 1560 = 0.01560 x 10° km/s Obviously vp = 0.65146 x 10° km/secisthe probable value of vp so = 0.1535 radians/ km with n = 0 is the proper value. Example 8.67. An open wire line, 200 miles long, has Zo = (683 - j 138) ohms and P= 0.0074 + j 0.0356 per miles at 1000 Hz. The line is driven by a generator with an open circuit emf of 10V rms at 1000 Hz and an internal resistance of 500 ohms. If the line is terminated in its own characteristic impedance, determine the sending end current, voltage and power and also the receiving end voltage, current and power. Solution. The equivalent sending end circuit for a line terminated in Zp is shown in Fig. 8:35. —-+I, Z5=2Q TRANSMISSION LINES AND MATCHING NETWORKS 695 Given Vz = 10 Volts; Zp=500 ohms ; Zs = Z) =683-j 138 ohms ; f = 1000 Hz Hence, sending end current /s is ge [oe | 10 _ 10 10 SS | Ze + Z| | $00 + 683 ~ j138 | ~ | 1183-7 138 | ~ | Vorr83 + (38 10 10 io =| Tisspagos Toad | ~ VTS = Troop ~ 0.008396! Amp. = 8.3961 m A (rms) Ans. Sending end voltage Vs is Vs= | Is Zs |=| 8.3961 x 10° x ( 683 —j 138 ) |=|8.3961 x 10° x (683 + (138) | = [8.3961 x 10° x ¥466489 + 19044 | = |8.3961 x 10° x ¥485533 | 8.3961 x 10° x 696.80] = |5850.4024 x 10°| = 5.8504024 Volts (rms) Vs = 5.85 Volts (rms) Ans. Average power entering the line is given by | Ps|=| Is P Rs= | (8.3961 x 10°)? | x 683 = 70.4945 x 10° x 683 = 48147.74 x 10° Watts = 48.14774 x 10 Watts = 48.147 milli-watts Ans. so Rs = 683 Q Now, if! is length of the line then by eqn. 8.30 voltage at receiving end is given by Vp = Vs e7™! aVEVe 3 eae Since 1 = 200 miles P =a. + jB = 0.0074 + j 0.0356 (given) or a = 00074 and B = 0.0356 Hence Va = Vse7(81I9! = Vs 08x eIP= 5,85 x eR *200 9) 00386 200 Vr = 5:85 x eM x eI? ‘Now from ‘“Exponential and hyperbolic functions”, table if If x=l4 then e7'® = 0.2466 If x=15 then! = 0.2231 Interpolating, we have 1.40 = 0.2466 1.50 = 0.2231 0.10 = 0.0235 or 0.01 = 0.00235 0.02 = 0.00235 x 2 = 0.00470 Hence 1.48 = 0.2231 + 0.0047 = + 0.2278 Hence eM = 0.2278 and Vpis given by Ve = 5.85 x 0.2278 Z - 7.12 x 57.3° = 1.33263 2 -407.976° = tama Z-407.976 + 2m = 1.33263 Z-47.976" = 1.33263 2-48" 1.33263, 133263 _ 99919125 Amps = 1.9125 mA Ans. “Ts + (138) 696.8 ELECTROMAGNETIC FIELOS AND WAVES 696 Then, Pe = [IR] x Re = |1.9125 x 10° x 683 = | 3.6576 x 10°| x 683 = 2498.17 x 10 W = 2.4981 x 10° W = 2.4981 mW Ans. Example 8.68. A telephone line 10 km long has the following constants per loop km. R= 19 ohms; L=7.1 mH ; C=009UF; G = negligible. The line is short circuited at the far end and a p.d. of 10 volts at a frequency of 50002 % Hz applied at the sending end. Calculate the current at the far end and its phase with applied voltage, Solution, The general eqn. is given by I. = Is cosh Py — ¥ sinh Py Vs = Vs cosh Py — Is Z sinh P, or Z Let! = length of the line which is short circuited at far end i.e. at x=1; Vy = 0 and i = Ir (say) Then, the general eqns. becomes 0 = Vs cosh Pl — Is Zp sinh Pl and In = Is cosh Pl ~ YS sinh PL Z _ ¥s cosh Pl or Is = ont eof8.147 (@)] Putting in above eqn., we have _ Vscosh*Pl Vs _ _ Vs cosh? Pl — sinh? Pl Te = “7. sinh PL ~ 2) SNP = 7 sinh PL Ip = —S— Ik = 7, sinh Pi + cosh? x — sinh? x = 1 [8.147 (b)] Now @ = 2n x 520 ~ 5000 radians an Z=R+jol = 196 + j 5000 x 7.1 x 107 = 196 +7 355 « |Z |= VC96" + 355)" Z tant ae = V38416 + 126025 2 0.1811 = ¥3967625 Z 10.24° = 199.1889 Z 10.24° Now Y= G+ jC =0 + j 5000 x 0.09 x 10° = 0 + j 0.45, x 107 or LY |= V(OAS TORY 2 tan O88 © (045 x 10) Z tan! | ¥ | = 045 x 107 2 90° Hone % = 4 [z _ x 199.1889 210.24" _ at 7 10.24" - 90 y 045 x 107 Z 90° Ma26a2 x 10" L 2 / 79.16 = Vee Z Re = 665.3134 2-39.88" In order to calculate P P =VZXY = V199.1889 x 045 M10" 2+ 1028 = Vooseas 2 10024" i 2 TRANSMISSION LINES AND MATCHING NETWORKS 697 .299391 2 $0.12° = 0.299391 [ cos 50.12" + j sin 50.12") .299391 [0.6418 + j 0.7676] = 0.1921 + j 0.2298 =a +) 8 .1921 and B = 0.2298 2298 x 10 = 2.298 and = a 1 = 0.1921 x 10 = 1921 inh (+jB )1=sinh (a 1 + JBL) inh 0. L cosh (j Bl) + cosh 0! sinh (j B”) = sinh aL! cos 1+, cosh a. sin BI or But 2.298 radians = 2.298 x 57.3" = 131.67" and @l = 1.92 radians = 110.01° Thus sinh Pl inh (90 + 20.01°) cos (90 + 41.67°) + j cosh (90 + 20.01°) sin (90 + 41.67") Since sinhjBJ=jsinBI; cosh jBI = cosB/ From exponential and hyperbolic functions table, 1.90 = 3.2682 = sinh 1.90 ana 2.00 = sinh 2.00 Difference or .587 x 0.02 = 0.0774 Hence = 3.2682 0714 Similarly cosh 1.90 cosh 2.00 Difference or 0.01 = 3.445 0.02 or cosh 1.92 = 3.4117 + 0.02 x 3.445 = 3.4117 + 0.0689 cosh 1.92 then from above eqn., we have sinh] = sinh 1.92 (—sin41.67°) + j cosh 1.92 cos 41.67" = 3.3396 x (- 0.665) + j x 3.4806 x 0.7488 sinh Pl = — 2.2204 + j 2.6062 ot [sinh Pt] = (R04) (DOOD Zan? - 20S = ¥4.9301 + 6.7924 Z-tan™ — 1.1737 = V11.7225 Z -49.54° + 180° sinh Pl = 3.4238 Z 130.46" Ms 130.46 + 39.88 Z sinh Pl 665.3134 Z— 39.88" x 3.4238 2130.46 2277.9 = 0.00439 2 - 90.58" Amp. = 4.39 2-90.58" mA =44 2-90.58" mA Ans. ‘Thus current at the far end is 4.4 m A which is lagging by 90.58” w.r-t. applied voltage. Hence [p= 698 ELECTROMAGNETIC FIELDS AND WAVES Example 8.69. The measurement on a transmission line at 1000 Hz gave the following results, Zy = 710 Z -16° ohms ; & = 0.01 neper/m and B = 0.035 rad/m. a its and the phase velocity on the line. Celeulae the primary consian ” (aMIRTE ‘Transmission Lines and Networks, May 1977) -1 0.035 Solution. P = a + jB = 0.01 + j 0.035 = V(O01)' + (0.035) Ztan! “o> P=V0.0001 + 0.001225 Z tan'!3.5 = ¥0.001325 2 73.92" = 0.0364 < 73.92" Hence Z=R+jwL=Px Z=0.0364 Z 73.92° x 710 2 — 16° = 25.844 Z 73.92° - 16° = 25.844 [cos 57.92 + jsin 57.92] = 25.844 [0.5319 + j 0.8474] R+j@L = 13.7464 + {21.90 Equating we have R = 13.7464 = 13.75 Q Ans. @L = 21.90 21.90 _ _ or = 558 5 1000 = 0.003487 = 3.487 mH Ans. voc a P. ~ 00364 £73.92" Further, G + jac = 7 = Sete = 512 x 10% 289.92" = 51.2 x 10% [cos 89.92" + j sin 89.92" ] G+j@C=51.2x 10° [ 0.0023 +; 1.000 ] = 0.11776 x 10° +7 51.2 x 10% Hence G = 0.11776 x 10° mhos/m = 0.1178 1 mhos/m Ans. 0.0000512 2 73.92° + 16° and @C = 51.2 x 10% 51.2x 10% | 12 or C= 5g0 = 0.0081528 x 10° F/m = 8152.8 x 10° F/m = 8152.8 PF/m Ans. The phase velocity v, is given by © _ 6280 Y= B= 9,935 ~ 17942857 m/sec = 1.7942857 x 10° m/sec = 1.7943 x 10? km/s. Ans. Example 8.70. The characteristic impedance of a line is 728 Z — 15.5° ohms and its Propagation constant is 0.00875 + j 0.0291 per km. The line is 200 km long and is terminated in an ‘impedance of 400 245° ohms. Find the current through load ifthe sending end voltage is 1 volts ‘ (AMIETE Transmission Lines and Networks, May 1972) Solution. Given Zp = 728 2 -15.5°; Ze = 400 245"; Vs = 1 volts: Ip = 2 P. = 0,00875 + j 0.0291 ; 1 = 200 km Since P = (0.00875 + j 0.0291) or PI = (0.00875 + j 0.0291) x 200 = 1.75 + j 5.82 ‘The current Ie through the load 400 Z 45° is given by or and hence In order to calculate Vp TRANSMISSION LINES AND MATCHING NETWORKS 699 Va = 1x eS 4158) (yH18) y (1582 y Vs= 1 Volts From natural logarithm table x = 1.70 = 0.1827 Difference Interpolating, we have or 0.05 = 0.174 x 0.05 = 0.0087 Hence 1.70 = 0.1827 od 5.82 radian = 5.82 x 57.3" = 333.486" = 21. - 333.5" = 26.5° Putting in above eqn., we get Vp =0.174 x [ cos 26.5° ~ j sin 26.5° ] = 0.174 x [ 0.8949 — j 0.4462 ] = 0.1557 -j 0.0776 | Ve |=V (0.1557)? + (0.0776 )* 2 tare eens = 0.0242 + 0.0060 Z — tan" 0.4983 =V 0.0302 4-26.5°= 0.1737 2 - 26.5° 0.1737 Z~ 26.5" _ ose . Thus Je= “fog ggr = 0.004342 4 -26.5° - 45° = 0.4342 £-71.5" mA Ans. Example 8.71. The characteristic impedance of a certain line is 710 Z 14° ohms, and P = 0.007 + 0.028 radians per km. The line is terminated in a 300 ohms resistor. Calculate the input impedance of the line, ifits length is 100 km. (AMIETE Transmission Lines and Networks, May 1971, Bang. Univ. BE. July 1990) Solution. Given Z=710 214"; P=0.007 +0028 ; Ze=300Q;1=100km PI = (0.007 + j 0.028) x 100 = 0.7 + j28 Now the input impedance Zs is given by Zs = Zo [SSaie ae cosh Pl = cosh ( 0.7 + 2.8 ) = cosh 0.7 cosh (2.8 ) + sinh (0.7) sinh (j2.8 ) cosh PI = cosh 0.7 cos 2.8 + j sinh O7 sin2.8 Since cosh(A +B) = cosh A cosh B + sinhA sinh B coshjA = cosA and sinh(jA) = jsinA Hence Now and Putting the values from tables we have, = 1.2552 x cos2.8 x 57.3° + j 0.7586 x sin2.8 x 57.3" cosh Pi = 1.2552 x cos (90° + 70.44") + j 0.7586 x sin (90° + 70.44") = 12552 x (sin 70.44") + j0.7586 x cos ( 70.44") 1.2552 x (-0.9425) + j0.7586 x (0.3344) cosh Pl = - 1.1830 + j 0.2536 = sinh (0.7 + j 2.8)= sinh 0.7 cosh (j2.8) + cosh 0.7 sinh (j2.8) sinh 0.7 x cos 2.8 + jcosh0.7 x sin 2.8 0.7586 x cos 160.44° + j 1.2552 x sin 160.44° = 0.7586 (—sin 70.44") + j 1.2552 x cos ( 70.44") = 0.7586 (- 0.9425) + j 1.2552 x 0.3344= - 0.7149 +; 0.4197 Now sinh PI 700 ELECTROMAGNETIC FIELDS AND WAVES Also Z is giventobe Z) = 710 214° = 710 [cos 14° + j sin 14°] Zp = 710 [0.9703 + j0.2419 ] = 688.913 + j 171.749 Now putting all the values to the eqn. of input impedance we get, the result i.e. = 210 2 14" [300 (- 1.1830 + j 0.2536) + {( 688.913 $j 171.749 ) (= 0.7149 +j0.4197)}) [( 688.913 +) 171.749) (- 1.1830 +] 0.2536) + [(— 0.7149 + 0.4197 ) 300] = 110 214" [354.9 + 776.08) - 492.50 — j 289.136 — j 122.783 +P 72.08) [= 814.9840 + j 174.708 - j 203.179 + 7? 43.55 - 214.47 + j 125.91] -1 242.43 +| (919.48)? + (7242.43) Z tan t _ Tio 2 14° [919.48 + j242.433)_10<"4 [ comsaBy' +0 = 919.48 = = ~1 97.439 [TE EET (HOTS + (97439 FZ tan 0 og _ TO 214" [VB45443.47 + 5877230 Z tant! (— 0.1059) 7 11151329 + 9494.3587 Z tan’ (— 0.0908 ) _ No 214° [Y¥90RIS.77_ 2-63] _ N10 214" [950.9026 2-63" ¥1160823,3 2-52 [1077.4151 2-5.2°] Zs _ 110 x 950.9020 4 14*-6.3° + 5751.40.84 Z 12.9° ~ 107.4151 ~~ 1077.4151 626.6302 2 12.9° ohms Ans. Zs Example 8.72. The primary constants for a certain transmission line operating at 7.5 kHz are : R= 26 ohms/loopkm ; L = 2.4 H/loop km C= 0.0078 p F/loop km ; G=0.11 1 Q /loop km. At the sending end of 50 km length of such a transmission line, an ideal generator having a voltage 10 V r.m.s. is connected and the output end is terminated in a matched load. Calculate the power consumed in the load. (AMIETE Transmission Lines Networks Noy. 1974) Solution. Z = R + j@L = 2.6 + j 6.28 x 7.5 x 10° x 24 x 10° = 2.6 + j 113.04 [Z| = V(26) + (113.04) Z tant 1B04 | Ve76e 12778.04 Z tan” 43.476 = V12784.801 2 88,7" = 113.069 Z 88.7" Y= G+ j@C = 0.11 x 10° + j 628 x 75 x 10° x 0.0078 x 10% 10 [0.11] + 0.36738 x 10° = 10% [0.11 + 7 367.38] | ¥ | = 10% V(O11) + (36738) Z tant 36138 | ¥ | = 10% V[O.0121 + 13496806] Z tan! 3539.8 = 10° 1134908.07 2 90" | ¥ | = 367.38 x 10% 2 90° Zz 113.09 2 88.7" Hence Z = WZ = V—11309 2887" _ apace 7 BO Y 367.38 x 10% 290° 03078 x 1 Z 2 = 0.554822 x 10° 2-0.65"= 554.82 Z-0.65° ohm. Also P= VZX ¥ = V113.09 288.7" x 367.38 x 10% 290" 178.7" = V41547.004 x 10% 2 J 203.83 x 10° 2 89.35° = 0.2038 2 89.35" ohms P= 0.2038 [cos 89.35 + j sin 89.35] = 0.2038 [0.0107 + j 0.9999] = (0.00218 + j 0.2037) TRANSMISSION LINES ANO MATCHING NETWORKS 701 Pl = (0.00218 + j 0.2037) x 50 = 0.109 + j 1.0185. Since line is terminated in a matched load so Zp = Zn of Input current or sending end current is Js = - 50km ; Vs=10V ;Z=% = Vs_ 10 7 Za SAB 2 COGS = 0.01802 2 +0.65" Amps or Is Now by definition of Propagation constant, ¢”! = & Ie or Ik = Is e™ = 0.01802 2 0.65" x e- (1% + 310185) ~ Qoig9r 20.65" x 7 (019 +4 10) _ 8 xe e108 ye g/L = 0.01802 2 0.65" x e xes ++ e710 = 0.9048 Fm table = 0.01802 20.65" x 0.9048 x [cos 1 x 57.3" — jsin 1 x 573°] .01802 4 0.65" x [0.4887 ~ j 0.7613] = (0.0088 - j 0.0137) Z 0.65" = WOOO F COIS & ta? = 0.0137 | te] = ¥(O0088 "+ (0.0137 2 tant SOE Pl + 0.65" | Ie | = ¥0.0000774 + 0.000169 Z tan”! — 1.5568 + 0.65" Y¥0.0002464 2 — 57.3° + 0.65" = 0.01569 Z - 57.3 + 0.65 = 0.01569 2-56.65". Hence the Power consumed in the load is given by Pa =| Ie f x Re = 0.0002461 x 554.82 = 0.13654 Watts where Rp is Resistive load of Zp Pr = 136.54 mW Ans, Example 8.73. A lossless transmission line having a characteristic impedance of 50 ohms (resistive) and of length 50 metres is left open circuited at the far end. If the open-circuit voltage is 100 20° volts. Calculate the voltage and the current at a distance of 10 metres away from the open circuit, assuming the generator frequency to 20 MHz. (AMIETE Transmission Lines and Networks, Nov. 1974) Solution. Given a = 0 as the line is lossless ZH =502;1=50m;y= 10m Ix = 0 as the far end is open circuited and no current flows f= 20 MHz and Vp = 100 £0" Volts then 4 = 2215 m and hence pete ss = 0.4186 radians = 0.4186 x 57.3° = 23.98" (degrees ) Now By = 23.98 x 10 = 239.85" If the distance is measured from the receiving end, the general eqns. of voltages and currents can be shown to be V = Vp cosh Py + Ip Z sinh Py ~-{8.148 (a)] I Ve In cosh Py + ZX sinh Py [8.148 (b)] “here ys the distance measured from receiving end. Since P=0+/jB=J8 for lossless line

You might also like